MBE Questions Flashcards

You may prefer our related Brainscape-certified flashcards:
1
Q

By federal statute, it is unlawful to knowingly possess any endangered species of fish or wildlife. Any person who violates this statute is guilty of a misdemeanor and is subject to a fine of not more than $50,000 or imprisonment of not more than six months, or both. Congress indicated that the statute should be applied as broadly as possible to accord the endangered species maximum protection.

Which of the following is likely the minimum that the prosecution must establish in order to obtain a conviction of a defendant found in possession of an endangered bird?

A) The defendant knew that the bird in her possession was a member of an endangered species.
B) The defendant knew that there was a federal statute that made possession of the bird a crime.
C) The defendant knowingly possessed the bird.
D) The defendant possessed the bird.

A

C) The defendant knowingly possessed the bird.

Most criminal offenses have a requisite mens rea, but courts may need to look at the statute’s legislative intent to appropriately apply this requirement.

AND the prosecution must prove that the defendant, at a minimum, possessed the requisite mens rea to obtain a conviction.

How well did you know this?
1
Not at all
2
3
4
5
Perfectly
2
Q

A convenience store clerk was complaining about his financial troubles to his best friend. The friend said that the clerk’s employer had been cheating the clerk out of a decent salary for too many years, and that the employer owed the clerk. The friend suggested that if the clerk robbed the store during another clerk’s shift, he would never get caught. The friend offered to loan the clerk his gun to use to scare the clerk on duty. Both men agreed that no one would get hurt in the process. The next day, the clerk carried out the plan to rob the store while the friend waited outside in a car. During the robbery, the clerk accidentally discharged the gun, and a customer was shot and died instantly. The clerk panicked and left the store empty-handed. The friend drove the clerk back to his mother’s house, told him to lay low, and then drove home. The clerk later decided that he needed to get out of town quickly. He stole his mother’s car, which was more reliable than his own, to drive to a nearby state.

In a case against the friend, which of the following charges would most likely be successfully prosecuted?

A) Attempted robbery, murder, and larceny.
B) Felony murder.
C) Attempted robbery only.
D) Attempted robbery and murder only.

A

B) Felony murder.

An accomplice is liable for
(1) the crime for which he/she provided encouragement or assistance and
(2) other crimes committed by the principal that were a natural and probable consequence of the accomplice’s conduct.

The friend, as an accomplice, is liable for crimes committed by the clerk that were a natural and probable consequence of the encouraged robbery, which includes felony murder. Therefore, the friend could be successfully prosecuted for felony murder—but not attempted robbery since it would merge into felony murder

How well did you know this?
1
Not at all
2
3
4
5
Perfectly
3
Q

A mother with a terminal illness told her son that she was in a great deal of pain. She requested that he end her suffering by taking her life. After unsuccessfully attempting to dissuade his mother, the son, who had no medical training, researched the matter on the Internet. Acting in compliance with his mother’s request, the son injected his mother with drugs that resulted in her quick and painless death. He did not financially benefit from his mother’s death in any way, as she had properly devised all of her property to a charity.

Of the following crimes listed in descending order of seriousness, which is the most serious crime of which the son may be convicted?

A) Murder.
B) Voluntary manslaughter.
C) Involuntary manslaughter.
D) Assisted suicide.

A

A) Murder
Common law murder is the unlawful killing of another with malice aforethought, which can be established by showing that the defendant acted with the intent to kill. Consent is never a defense to this crime.

How well did you know this?
1
Not at all
2
3
4
5
Perfectly
4
Q

A carpenter was using a nail gun in the construction of a small outdoor deck on a house. The carpenter knew that the nail gun could fire a nail with sufficient force to kill a human being at close range. Aware of the presence of three other carpenters on the other side of the nearly finished deck, one of whom the carpenter disliked, the carpenter fired the nail gun twice at a table located between himself and the other three carpenters. The first nail from the gun struck the table but the second nail, ricocheting off the table, struck and killed the disliked carpenter. At trial, a jury, based on this evidence, found the carpenter guilty of murder.

If the carpenter appeals his conviction on the ground that the evidence was not sufficient to support his conviction, how should the appellate court rule?

A) Affirm the conviction, because the evidence is sufficient to establish that the carpenter acted with criminal negligence.
B) Affirm the conviction, because the evidence is sufficient to establish that the carpenter acted with malice aforethought.
C) Overturn the conviction and remand for a new trial, because the evidence is not sufficient for murder but can support an involuntary manslaughter conviction.
D) Overturn the conviction and remand for a new trial, because the evidence is not sufficient for murder but can support a voluntary manslaughter conviction.

A

B) Affirm the conviction, because the evidence is sufficient to establish that the carpenter acted with malice aforethought.

Common-law murder is the unlawful killing of another with malice aforethought. This include depraved-heart murder.

Here, the evidence is sufficient to establish that the carpenter committed depraved-heart murder when he fired a nail gun in the direction of the other carpenters who were standing close by. That is because he knew that the nail gun could generate enough force to kill a human being, thereby disregarding an obvious risk of causing death or serious bodily injury when he fired it.

How well did you know this?
1
Not at all
2
3
4
5
Perfectly
5
Q

The defendant, his brother, and his best friend formed a plan to rob a bank. On the day of the crime, the defendant and his brother entered the bank carrying guns, while the best friend stayed in the car to act as a getaway driver. After the defendant received one bag of money from a bank teller, he saw a security guard pull out a gun. The defendant tried to shoot the security guard but instead shot his brother. The defendant panicked and ran out of the building toward the getaway car. The security guard chased the defendant and fired a shot toward him as he approached the car. The shot hit the best friend, who was in the driver’s seat of the car. Shortly thereafter, the police arrived and arrested the defendant. The brother and the best friend later died as a result of their gunshot wounds.

For which of the following crimes is the defendant most likely to be convicted and punished?

A) Robbery and two counts of felony murder for the death of the brother and the best friend.
B) Robbery and one count of felony murder for the death of the brother.
C) Two counts of felony murder for the death of the brother and the best friend.
D) One count of felony murder for the death of the brother.

A

D) One count of felony murder for the death of the brother.

In most jurisdictions, the underlying felony (here, robbery) merges into felony murder. This means that a defendant convicted of felony murder cannot also be convicted of the underlying felony.

How well did you know this?
1
Not at all
2
3
4
5
Perfectly
6
Q

After a woman and her roommate got into a fight, the woman and her boyfriend formulated a plan to get back at the roommate. The woman planned to distract the roommate while the boyfriend would sneak into the apartment through the open back door to steal her new camera, which was on the kitchen counter. The woman and her boyfriend headed back to the apartment and put their plan into action. However, the plan went awry when the roommate heard a noise and went into the kitchen to find the boyfriend with her camera in hand. Panicked, the boyfriend pushed the roommate to the floor and ran out of the apartment with the camera.

The boyfriend is most likely to be convicted of which of the following crimes?

A) Burglary only.
B) Burglary and battery.
C) Robbery only.
D) Robbery and battery.

A

C) Robbery only

Robbery is larceny from the victim’s person or presence by force (battery) or intimidation (assault).

Merger: Robbery + Battery
Since larceny and battery/assault are lesser included offenses to robbery, they merge into the completed robbery. Therefore, the defendant may only be convicted of robbery—not its constituent offenses.

How well did you know this?
1
Not at all
2
3
4
5
Perfectly
7
Q

A woman broke into her former lover’s house at night with the intent to take back various items of her clothing that the former lover had refused to return to her. After conducting a search and being unable to find the clothing, she came across another woman’s clothing. Extremely angry, she took a cigarette lighter and lit the lover’s bed on fire, destroying the bed.

With which of the following crimes can the woman be properly charged?

A) Arson.
B) Burglary.
C) Both arson and burglary.
D) Neither arson nor burglary.

A

D) Neither arson nor burglary

Common-law arson = requires fire damage to the actual building structure—not just the contents of the dwelling.

Here, the woman burned only the former lover’s bed—not a part of the actual structure of the building—so she cannot be properly charged with arson

Common-law burglary = the requisite intent must exist at the time of entry.

Here the woman broke and entered the house of her former lover at night to recovery her own clothing—not to commit a felony therein.

How well did you know this?
1
Not at all
2
3
4
5
Perfectly
8
Q

A) An attorney represented a client in a legal battle over a valuable necklace that had belonged to the client’s deceased grandmother. The attorney told the client, who had possession of the necklace, that the client was legally required to leave the necklace with the attorney until the legal issues were resolved. In fact, there was no such requirement. Rather, the attorney intended to sell the necklace and retire on a small island where the attorney believed she would never be found. After the client gave the necklace to the attorney, the attorney sold it to a jeweler. The jeweler, who had known the grandmother, later recognized the necklace as the grandmother’s and called the police. The attorney was arrested at the airport later that day.

The attorney is guilty of which of the following crimes?

A) Embezzlement.
B) False pretenses.
C) Larceny by trick.
D) Robbery.

A

C) Larceny by trick

Larceny by trick is a larceny accomplished by fraud or deceit that results in the conversion of the property of another. Larceny by trick differs from false pretenses in that the defendant acquires mere possession of (not title to) the property.

How well did you know this?
1
Not at all
2
3
4
5
Perfectly
9
Q

A woman broke into her ex-husband’s house late one night when she knew he was away on business, intending to take a sculpture that he had been awarded in their divorce settlement. She searched the entire house but was unable to find the sculpture. She figured he had probably sold the sculpture, which made her furious because he knew how much she loved it. In a rage, she slashed a painting of his new girlfriend before leaving the house.

Of which of the following crimes is the woman guilty?

A) Attempted larceny only.
B) Larceny only.
C) Burglary only.
D) Burglary and attempted larceny.

A

D) Burglary and attempted larceny.

Common law burglary = complete when the defendant unlawfully breaks and enters a dwelling at night with the intent to commit a felony therein.
+ Commission of the underlying felony is unnecessary.
+ BUT a burglary defendant who fails to complete the underlying felony is also guilty of the attempted commission of that felony

Here, the woman is guilty of burglary because she broke into her ex-husband’s home at night with the specific intent to commit larceny. Although the woman did not complete the larceny because she did not actually steal the sculpture, she is still guilty of attempted larceny

How well did you know this?
1
Not at all
2
3
4
5
Perfectly
10
Q

A woman searched on the Internet for a hit man who would kill her husband in exchange for cash. She found a purported mercenary in a chat room who agreed to kill the husband in return for $50,000. The mercenary turned out to be an undercover police officer, and the woman was arrested.

With which of the following common law crimes could the woman properly be charged?

A) Conspiracy only.
B) Solicitation only.
C) Both conspiracy and solicitation.
D) Neither conspiracy nor solicitation.

A

B) Solicitation only.

CL Conspiracy = requires proof of at least two guilty minds, so a defendant cannot be convicted of conspiracy if the other alleged conspirator(s) feigned agreement.

Solicitation = complete upon the encouragement of the crime—it does not matter if the other person agrees to or can commit the solicited crime.

How well did you know this?
1
Not at all
2
3
4
5
Perfectly
11
Q

A daughter was homeschooled by her bohemian parents for most of her life. The parents did not believe in mainstream medicine, so they taught the daughter about homeopathic remedies as part of her homeschooling. She helped her parents with their extensive garden as part of her daily chores. Unfortunately, the daughter was lonely, she did not believe in homeopathy, and she wanted to eat meat, so she resolved to kill her parents by poisoning them but making it look like an accident.

Among the plants the daughter was growing was the castor oil plant, which her parents used to treat a variety of skin conditions and to stimulate the immune system. However, the seeds of the castor oil plant contain the toxin called ricin, which is deadly if ingested by humans. The daughter extracted the seeds from a castor oil plant, crushed them, and then added the ricin powder to her parents’ oatmeal. Unbeknownst to the daughter, she had actually picked the wrong plant, known as the “false castor oil plant,” which was similar in appearance but did not produce poisonous seeds. After the parents ate the oatmeal laced with the harmless seeds, they suffered some gastric distress. They went to the garden to find an herbal remedy, and noticing that all of the false castor oil plants were missing, they suspected that someone untrained in plant species had tried to poison them, so they called the police. The police were able to piece together what actually happened, and they arrested the daughter for attempted murder.

Could the daughter be found guilty of attempted murder?

A) No, because factual impossibility is a defense to the crime of attempted murder.
B) No, because it was legally impossible for the daughter to kill her parents.
C) Yes, because mistake of fact is not a defense to the crime of attempted murder.
D) Yes, because factual impossibility is not a defense to the crime of attempted murder.

A

D) Yes, because factual impossibility is not a defense to the crime of attempted murder.

Attempt occurs when a person
(1) takes a substantial step toward the commission of a crime
(2) with the specific intent to commit the crime.

Factual impossibility is never a defense to attempt.

How well did you know this?
1
Not at all
2
3
4
5
Perfectly
12
Q

Three coworkers were employed at a jewelry store. They were always complaining to one another about the long hours, the paltry pay rate, and the lack of medical benefits. Two of the coworkers came up with a plan to steal a valuable collection of gems from the jewelry store, but it was a three-person job; they asked the third coworker to join in on their plan, but he refused. The two coworkers knew how devoted the third coworker was to his family, so they threatened to kill his wife and kids if he did not help them or if he tried to foil their plan by notifying the police. Believing that he had no other choice, the third coworker joined in on the plan.

The three men entered the jewelry store that night, having received the permission of their boss in order to polish all of the jewelry in preparation for a jewelry show the next day. While the two coworkers broke into the wall safe where the gems were kept, the third coworker kept watch for the night guard on his hourly walk-through of the store premises. The night guard decided to do his walk-through 10 minutes early, saw the men breaking into the safe, and pulled out his gun. The excitement of the event caused the night guard to have a heart attack, which was fatal. The police entered at that moment and arrested the three men for burglary and felony murder of the night guard.

What is the third coworker’s best defense against the felony-murder charge?

A) He did not intend to kill the night guard.
B) He was under duress to commit the burglary.
C) He had permission to enter the jewelry store.
D) He could not have foreseen the security guard’s heart attack.

A

B) He was under duress to commit the burglary.

Duress is a defense to felony murder when a third party’s unlawful threat causes a defendant to reasonably believe that the only way to avoid death or serious bodily injury to himself or another is to violate the law—thereby causing the defendant to do so.

Note: If entry is gained with consent, a breaking can still occur if the defendant breaks into a part of the structure—e.g., by opening a closet door or wall safe (as the coworkers did here). Therefore, the fact that the third coworker had permission to enter the jewelry store is also no defense

Note: Duress is not a defense to intentional murder

How well did you know this?
1
Not at all
2
3
4
5
Perfectly
13
Q

Dusty and Walt were coworkers. Dusty admired Walt’s wristwatch and frequently said how much he wished he had one like it. Walt decided to give Dusty the watch for his birthday the following week. One the weekend before Dusty’s birthday, Dusty and Walt were at a company picnic. Walt took off his watch to join a softball game. Dusty strolled by, saw the watch on the blanket and decided to steal it. He bent over and picked up the watch. Before he could pocket it, however, Walt returned. When he saw Dusty holding the watch, he said, “Dusty, I know how much you like that watch. I was planning to give it to you for your birthday. Go ahead and take it now.”

What crime, if any, has Dusty committed?

A) Attempted Larceny
B) Larceny
C) Embezzlement
D) No crime

A

B) Larceny

Once Dusty bent over and picked up the watch he exerted control over the object and possessed the requisite intent.

How well did you know this?
1
Not at all
2
3
4
5
Perfectly
14
Q

Hink, a college student, was a member of a fraternity. On Saturday night, Hink and his friend Lee were attending a toga party at the fraternity house. While the party was in progress, a few students from a rival fraternity vandalized some of the cars parked outside the fraternity house. They broke the headlights and stole the battery from Hink’s car. When the party ended, Hink and lee left the fraternity house and got into his car. Hink, who was about to drive Lee home, was unaware of what happened. He tried to start the car, but it wouldn’t turn on. Two police officers, who were parked outside the fraternity house, watched Hink as he tried to start the car. They then approached Hink and charged him with attempting to violate a local ordinance making it a misdemeanor crime to drive at night without headlints.

What is Hink’s best defense to the charge?
A) Factual Impossibility
B) Legal impossibiility
C) Entrapment
D) no requisite intent

A

D) no requisite intent

Attempt crimes = Specific intent (fiAt)

Hink had no knowledge of the fact that his headlights were out.

How well did you know this?
1
Not at all
2
3
4
5
Perfectly
15
Q

A state statute provides: “The sale of an alcoholic beverage to any person under the age of 21 is a misdemeanor.” A woman who was 20 years old but looked older and who had a very convincing fake driver’s license, indicating she was 24, entered a convenience store, picked up a six-pack of beer, and placed the beer on the counter. The store clerk, after examining the driver’s license, rang up the purchase. Both the clerk and store owner have been charged with violating the statute.

If the court finds both the clerk and the store owner guilty, what standard of liability must the court have interpreted the statute to impose?

A) Strict liability only
B) Vicarious liability only
C) Both strict and vicarious liability
D) Either strict or vicarious liability

A

C) Both strict and vicarious liability

Statutory liability = mens rea not considered.

Vicarious liability: clerk was acting within the scope of his employment

How well did you know this?
1
Not at all
2
3
4
5
Perfectly
16
Q

A defendant was validly arrested for the murder of a store clerk and was taken to a police station where he was given Miranda warnings. When an interrogator asked the defendant, “Do you understand your Miranda rights and are you willing to give up those rights and talk with us.” The defendant replied, “Yes.” When asked, “Did you kill the clerk, the defendant replied, “No.” When asked, “Where were you on the day the clerk was killed?” the defendant replied, “Maybe I should talk to a lawyer.” The interrogator asked, “Are you sure?” and the defendant replied, “I’m not sure.” The interrogator then asked, “Why would you want to talk with a lawyer? and the defendant replied, “because I killed the clerk. It was an accident, and I think I need a lawyer to defend me.” At that point all interrogation ceased. Later, the defendant was formally charged with murdering the clerk.

The defendant has moved to suppress evidence of his statement, “I killed the clerk” on the grounds that this statement was elicited on the violation of his Miranda Rights.

Should the defendant’s motion be granted?

A) No, because although the defendant effectively asserted the right to counsel, the question, “Why would you want to talk with a lawyer?” did not constitute custodial interrogation.

B) No, because the defendant did not effectively assert the right to counsel and his conduct prior to making the statement constituted a valid waiver of his Miranda rights.

C) Yes, because although the defendant did not effectively assert the right to counsel, his conduct prior to making the statement did not constitute a valid waiver of his Miranda rights.

D) Yes, because the defendant effectively asserted the right to counsel and the question, “Why would you want to talk with a lawyer?” constituted custodial interrogation.

A

B) No, because the defendant did not effectively assert the right to counsel and his conduct prior to making the statement constituted a valid waiver of his Miranda rights.

Asserting rights to lawyer: must be clear and unambiguous

The defendant waived his rights when he said he understood them and was willing to speak with police.

How well did you know this?
1
Not at all
2
3
4
5
Perfectly
17
Q

A wife decided to kill her husband because she was tired of his infidelity. She managed to obtain some cyanide, a deadly poison. One evening, she poured wine laced with the cyanide into a glass, handed to her husband and proposed a loving toast. The husband was so pleased with the toast that he set the glass of wine down on a table, grabbed his wife and kissed her passionately. She hid the glass of wine behind a lamp, planning to leave for the maid to clean up. The husband did not drink the wine. The maid found the glass of wine while cleaning the next day. Rather than throw the wine away, the maid drank it. Shortly thereafter, she fell into a coma and died from poisoning.

A) Attempted murder of the husband and murder or manslaughter of the maid
B) Only attempted murder of the husband
C) Only the murder or manslaughter of the maid
D) No crime

A

A) Attempted murder of the husband and murder or manslaughter of the maid

CL Murder: unlawful killing of another human with malice aforethought
+ Malice = intent to kill, intent to cause GBH, depraved heart

Attempt: Specific Intent crime + substantial attempt towards crime

Here, the wife had the requisite intent to kill the husband and took reasonable steps towards the crime, but the crime incomplete, thus making it attempted murder. Her intent can transfer to the maid and the crime was completed.

How well did you know this?
1
Not at all
2
3
4
5
Perfectly
18
Q

A woman broke off her engagement to a man but refused to return the engagement ring. One night, the man entered the woman’s house after midnight to retrieve the ring. Although the woman was not at home, a neighbor saw the man enter the house and called for the police. The man unsuccessfully searched for the ring for 10 minutes. As he was walking out the front door, the police arrived and immediately arrested him. The man has been charged with burglary in a jurisdiction that follows the common law.

Which of the following, if proved, would serve as the man’s best defense to the charge?

A) the man knew that the woman kept a key under the doormat and he used the key to enter the house
B) The man incorrectly and unreasonably believed that he was legally entitled to the ring
C) The man knew that no one was at home when he entered the house
D) The man took nothing of value from the house

A

B) The man incorrectly and unreasonably believed that he was legally entitled to the ring

Theft: Specific Intent Crim (fiaT), includes burglary

Defense to specific intent crime: belief, reasonable or unreasonable, and mistake of fact – negates mens rea of the intent

How well did you know this?
1
Not at all
2
3
4
5
Perfectly
19
Q

A defendant is on trial for knowing possession of a stolen TV. the defendant claims that the TV was a gift from a friend, who has disappeared. The defendant seeks to testify that he was present when the friend told her neighbor that the TV had been given to the friend by her mother.

Is the defendant’s testimony about the friend’s statement to the neighbor admissible?

A) No because the friend’s statement is hearsay not within any exception
B) No because the defendant has not presented evidence of circumstances that clearly corroborate the statement
C) Yes, as nonhearsay evidence of the defendant’s belief that the friend owned the TV
D) Yes, under the hearsay exception for statements affecting an interest in property

A

C) Yes, as nonhearsay evidence of the defendant’s belief that the friend owned the TV

Declarant’s State of Mind - we don’t need the statement to be true, just to show the defendant’s belief that the TV wasn’t stolen

How well did you know this?
1
Not at all
2
3
4
5
Perfectly
20
Q

The defendant was charged with conspiracy to distribute drugs for his alleged involvement in a major drug-trafficking ring. The evidence tying the defendant to the drug-trafficking ring was obtained from a warrantless search of the home of a business partner. The defendant had been living with his business partner but recently moved out, leaving a bag filled with cocaine and his identification in the business partner’s home. The bag was discovered by police during a search of the home with the business partner’s consent. Prior to trial, the defendant filed a motion to suppress the cocaine.

How should the court rule on the defendant’s motion to suppress?

A) Deny the motion, because the business partner consented to the search of the home.
B) Deny the motion, because the defendant does not have a privacy interest in the home.
C) Grant the motion, because the defendant had a reasonable expectation of privacy in the home.
D) Grant the motion, because the evidence was discovered as a result of a warrantless search.

A

B) Deny the motion, because the defendant does not have a privacy interest in the home.

4th Amendment challenge = unreasonable search and seizure
+ legitimate expectation of privacy
+Cannot claim privacy to a home where the defendant doesn’t live.

Note: To prevail on a motion to suppress, the defendant must establish that he has standing to contest the allegedly unlawful Fourth Amendment search. Standing exists when the defendant has a legitimate expectation of privacy (or an ownership/possessory interest) in the area or item searched at the time of the search.

It doesn’t matter if the biz partner consented to the search, we are focusing on the defendant’s standing/expectation of privacy

How well did you know this?
1
Not at all
2
3
4
5
Perfectly
21
Q

Two officers went to a man’s home to serve an arrest warrant, but no one answered the door. As they walked around the man’s house, the officers looked into his next door neighbor’s window and saw the man inside the neighbor’s kitchen drinking coffee. The officers knocked on the neighbor’s door, and when the neighbor answered, the officers informed the neighbor that they had a warrant to arrest the man. The officers pushed past the neighbor into the kitchen and arrested the man. While they were in the kitchen, the officers saw a bag of marijuana on the neighbor’s counter. The officers arrested the neighbor, and he was subsequently charged with possession of narcotics. The neighbor moved to suppress evidence of the drugs, and the prosecution argued that the evidence was admissible under the plain view exception to the warrant requirement.

Are the drugs seized in the neighbor’s kitchen likely to be admitted against the neighbor?

A) No, because the officers could not lawfully enter the neighbor’s home without his consent.
B) No, because the officers did not knock and announce to the neighbor their intention to arrest the man.
C) Yes, because the arrest warrant implicitly authorized the officers to take measures necessary to serve the warrant.
D) Yes, because the marijuana was in plain view when the police arrested the man.

A

A) No, because the officers could not lawfully enter the neighbor’s home without his consent.

An arrest warrant implicitly authorizes entry into the arrestee’s home—not a third party’s home—to serve the warrant. Police may only search for an arrestee in a third party’s home if they have a warrant for the search, exigent circumstances, or consent to enter.

Note: The plain-view doctrine allows an officer conducting a lawful search to seize an apparently illegal item in plain view—even if the item was not named in a warrant—if the officer has lawful access to the item (not seen here).

How well did you know this?
1
Not at all
2
3
4
5
Perfectly
22
Q

A police officer received an anonymous tip that the defendant was manufacturing methamphetamine in his basement. Based solely on the tip, the officer obtained a warrant to search the defendant’s basement for drugs and related manufacturing equipment. The officer and his partner went to the defendant’s home to execute the warrant. Believing the defendant was not home, the officers did not knock on the door, but simply opened the unlocked door. In searching the defendant’s basement, the officers found large quantities of methamphetamine, related manufacturing equipment, and a notebook that said “Ledger” across the cover. The notebook contained a ledger, with the names of the defendant’s clients and statements of their accounts. The officers seized all these items. The defendant seeks to suppress the evidence seized by the officers.

What is the defendant’s best argument in favor of suppressing the notebook?

A) The notebook was not named in the warrant.
B) The notebook was in the nature of a personal diary.
C) The officers failed to “knock and announce” their presence.
D) The warrant was invalid.

A

D) The warrant was invalid.

Probable cause to support a search warrant can come from information supplied by
(1) a *reliable, known informant or
(2) an unknown informant if the information is independently verified

Here, the police made no attempt to verify the tip from an unknown informant, so probable cause was not established. Here = invalid warrant due to lack of verification of anonymous tip.

Note: Had the warrant been valid, then the police could have lawfully seized any items named in that warrant (e.g., methamphetamine, manufacturing equipment). They could have also seized any item in plain view—even if they were not named in the warrant (e.g., the notebook)—if the incriminating nature of the item was immediately apparent and the officers had lawful access to it.

How well did you know this?
1
Not at all
2
3
4
5
Perfectly
23
Q

Two undercover police officers, with probable cause to believe that the defendant was a drug dealer, entered the living room of the defendant’s apartment, at the defendant’s invitation, to buy cocaine. Before the transaction could take place, the defendant shot and killed one of the officers. After a brief struggle, the defendant was subdued by the other officer and placed under arrest for murder. Responding to the officer’s request for assistance, uniformed police officers came to the apartment, conducted a protective sweep, and took the defendant to jail. Then the uniformed officers conducted a thorough warrantless search of the apartment, during which they uncovered a large quantity of cocaine in the mattress in the defendant’s bedroom. Based on the amount of cocaine seized, the defendant was charged with possession of cocaine with intent to deal in addition to murder. The defendant filed a motion to suppress the cocaine as having been unconstitutionally seized.

Should the court grant this motion?

A) Yes, because the search was conducted without a warrant.
B) Yes, because the defendant was arrested for murder, not drug dealing.
C) No, because the defendant had a lesser expectation of privacy once arrested.
D) No, because the murder created exigent circumstances.

A

A) Yes, because the search was conducted without a warrant.

Police may conduct a warrantless search incident to an arrest of the arrestee and the immediate surrounding areas. And the exigent-circumstances exception allows police to conduct a protective sweep to search for injured persons and continued threats.

Once the emergency ended—i.e., the defendant was restrained and no other threats were located—the police were not justified in continuing the warrantless search without an exception:

Exigent circumstance
Search incident to arrest
Consent
Automobile exception
Plain view
Evidence obtained from gov. purpose/admin search
Stop and frisk

Mnemonic: ESCAPES

How well did you know this?
1
Not at all
2
3
4
5
Perfectly
24
Q

While on patrol one night, two officers noticed the car of a known drug dealer in the drive-through lane of a fast-food restaurant. Based on prior discussions with informants, the officers had probable cause to believe that the drug dealer had drugs in his vehicle since he regularly made drug deliveries from the trunk of his car. Noticing that the drug dealer’s headlight was out, the officers pulled him over once he left the restaurant and searched his car. The officers did not find any evidence of drugs, but they did find several illegal weapons in the trunk.

Did the officers’ seizure of the weapons violate the drug dealer’s Fourth Amendment rights?

A) Yes, because the drug dealer could not access the trunk from the passenger compartment.
B) Yes, because the officers were searching for drugs, not weapons.
C) Yes, because the stop was pretextual in nature.
D) No, as a valid application of the automobile exception to the warrant requirement.

A

D) No, as a valid application of the automobile exception to the warrant requirement.

SAD SPACES = exceptions to searches without warrant

Automobile: allows police to conduct a warrantless search of a vehicle if they have probable cause to believe it contains evidence of a crime.

Officers can search any area within the vehicle where the evidence might be located, including the trunk and locked containers.

Note: Limitation to areas that the driver can access = only when there is a search incident to a lawful arrest

How well did you know this?
1
Not at all
2
3
4
5
Perfectly
25
Q

A grand jury is investigating a corporation for tax fraud. The grand jury subpoenaed several types of corporate records, including emails from several of the corporation’s officers and other business papers. The subpoena was served on the corporation’s chief operating officer (COO), who acted as the custodian of the corporate records. The business papers would implicate several members of the corporation, including the COO, in criminal misconduct. Further, many of the emails written by the COO contained statements about the officers of some of the corporation’s competitors; these statements were defamatory and would likely result in civil liability if they were released. The COO filed a motion to quash the subpoena, arguing that being forced to turn over these corporate records would violate his Fifth Amendment privilege against self-incrimination.

Should the court grant the COO’s motion?

A) No as to both the emails and the business papers.
B) No as to the emails, but yes as to the business papers.
C) Yes as to the emails, but no as to the business papers.
D) Yes as to both the emails and the business papers.

A

D) Yes as to both the emails and the business papers.

Fifth Amendment: does not apply to evidence that might subject a person to civil liability or to corporations, individuals only

Here, since the privilege does not apply to corporations, the court should deny the COO’s motion to quash as to both the emails and the business papers even though they incriminate the COO

How well did you know this?
1
Not at all
2
3
4
5
Perfectly
26
Q

Following the armed robbery of a local bank, the police identified the defendant as a suspect and brought him to the police station for questioning. As soon as they sat down in the interrogation room, the police read the defendant his Miranda rights. The defendant noted that “this seems like the kind of thing where you should have a lawyer.” The police responded that the defendant had that right. The defendant noted that he “didn’t even know a good lawyer” and dropped the issue. In response to each question, the defendant simply repeated, “I don’t know anything about it.” Frustrated, the police discontinued questioning after an hour and left the defendant in the interrogation room alone. Three hours later, the police returned and, without repeating the Miranda warnings, told the defendant that his best friend, who was also a suspect, had already told them all about the robbery and the defendant’s involvement. In fact, the police were searching for the defendant’s best friend to bring him in for questioning but had not been able to locate him. The defendant immediately blurted out, “It was all his idea. I didn’t even want to rob that bank.”

What is the defendant’s best argument that his statement was taken in violation of his Fifth Amendment rights?

A) The defendant did not receive fresh Miranda warnings after the break in questioning.
B) The defendant did not sign a written waiver of his Miranda rights.
C) The lie regarding his best friend’s statement rendered the defendant’s statement involuntary.
D) The police continued to question the defendant after he invoked his right to counsel.

A

A) The defendant did not receive fresh Miranda warnings after the break in questioning.

Custodial interrogation: in custody, subject to interrogation

If the interrogation of a suspect who has waived his/her Miranda rights is stopped for a long duration, police should re-Mirandize the suspect prior to resuming the interrogation.

How well did you know this?
1
Not at all
2
3
4
5
Perfectly
27
Q

A defendant was arrested and charged with robbery. While he was awaiting trial, an inmate in the jail in which he was housed was assaulted, and the police suspected that the defendant was involved. They brought the defendant in for questioning about the assault and provided him with Miranda warnings. The defendant said that he was willing to talk and did not ask for his attorney. He proceeded to tell the police that he had provided another inmate with information about how to obtain a weapon and believed that inmate had been involved in the assault. The defendant was later charged as a co-conspirator in the assault and sought to suppress his statement to the police. He argued that his attorney should have been present during the interrogation.

Is the defendant’s statement likely to be suppressed?

A) No, because the defendant did not specifically invoke his Fifth Amendment right to counsel.
B) No, because the defendant waived his Sixth Amendment right to counsel by failing to request his attorney.
C) Yes, because the defendant did not waive his Fifth Amendment right to counsel.
D) Yes, because the defendant did not provide a knowing, voluntary, and intelligent waiver of his Sixth Amendment right to counsel.

A

A) No, because the defendant did not specifically invoke his Fifth Amendment right to counsel.

5th Amendment right to counsel: have to invoke right to counsel

6th Amendment right to counsel: do not have to invoke, applies automatically once commencement of judicial proceedings

Pay attention to the crimes and stage of charges

How well did you know this?
1
Not at all
2
3
4
5
Perfectly
28
Q

Two police officers responded to a domestic-disturbance call. When the officers arrived at the home in question, they found a man and a woman screaming at each other in the front yard. One officer took the man aside, while the other spoke with the woman. The woman told the officer that she had a restraining order against the man and that he was a convicted felon who often carried a gun. The officers confirmed the restraining order and placed the man under arrest, without giving him Miranda warnings. While the man was in the back of the police car, an officer asked the man if he owned a gun. The man replied that he always keeps a gun in his car for self-protection. The officer then retrieved the gun from the man’s car, which was parked nearby.

The man was later charged with possession of a firearm as a felon. At trial, the man moved to suppress his statement about always having a gun and the gun itself, arguing that he did not receive Miranda warnings prior to being questioned by the officer.

Is the man likely to succeed in having the evidence suppressed?

A) No, as to both the statement and the gun.
B) Yes, as to the statement only.
C) Yes, as to the gun only.
D) Yes, as to both the statement and the gun.

A

B) Yes, as to the statement only.

A suspect’s incriminating statement during a custodial interrogation without Miranda warnings cannot be used against the suspect at a subsequent trial.

BUT physical evidence obtained as a result of the non-Mirandized statement is admissible so long as that statement was not coerced.

How well did you know this?
1
Not at all
2
3
4
5
Perfectly
29
Q

A man, who was represented by counsel, was on trial for arson of an office building. During the trial, the police learned that computers thought to have been destroyed in the fire had instead been stolen. The police visited the man in jail, read him his Miranda rights, which the man voluntarily waived, and questioned him about the theft. In response to the questioning, the man confessed to the theft.

After the man was found not guilty of arson, the prosecutor charged the man with theft and informed the defense that he planned to introduce the man’s statement to the police while he was in jail. The man’s attorney filed a pretrial motion to suppress the statement on the ground that the man’s Sixth Amendment right to counsel had been violated.

How should the court rule on this motion?

A) Deny the motion, because the man was read and waived his Miranda rights.
B) Deny the motion, because theft and arson each require proof of an element that the other does not.
C) Grant the motion, because the arson and the theft took place during the same criminal transaction.
D) Grant the motion, because the defendant was represented by counsel when he was questioned about the theft.

A

B) Deny the motion, because theft and arson each require proof of an element that the other does not.

6th Amendment right to counsel: attaches to an uncharged crime that constitutes the same offense as a formally charged crime.

DOES NOT attach to an uncharged crime that requires proof of an element that the other does not.

Here, the man had been formally charged with arson at the time of the interview, so he had a Sixth Amendment right to counsel for that crime. However, he had not been charged with the theft. And since arson and theft each require proof of an element that the other crime does not, the man had no Sixth Amendment right to counsel for the theft. Accordingly, the court should deny the motion to suppress the man’s statement.

How well did you know this?
1
Not at all
2
3
4
5
Perfectly
30
Q

A defendant was indicted by a grand jury for making false statements to a federally insured bank to obtain a loan in violation of a federal felony statute. After the defendant was arraigned, the federal district court granted the defendant’s request for disclosure of the exculpatory portions of the grand jury transcripts. When the defendant learned that the prosecutor had failed to present the defendant’s federal tax returns in the prosecutor’s possession to the grand jury even though the returns contained exculpatory evidence, the defendant moved to dismiss the indictment.

How should the court rule on the defendant’s motion?

A) Deny the motion, because the defendant knew of his own tax returns.
B) Deny the motion, because the prosecutor was not required to present exculpatory evidence to the grand jury.
C) Grant the motion, because the prosecution of a felony by the federal government must be based on a proper indictment.
D) Grant the motion, because the prosecutor withheld Brady material.

A

B) Deny the motion, because the prosecutor was not required to present exculpatory evidence to the grand jury.

Grand jury: A prosecutor need not present exculpatory evidence to a grand jury that is considering whether to indict a person.

This evidence is required after indictment and prior to trial.

How well did you know this?
1
Not at all
2
3
4
5
Perfectly
31
Q

A defendant on trial for burglary planned to raise in his defense that his arrest was a result of racial profiling. During voir dire, the prosecutor exercised his peremptory challenges to exclude all nonwhite jurors, not just those of the defendant’s race, from the jury panel. When the defendant objected to these challenges as discriminatory, the prosecutor responded that he was concerned that nonwhite jurors would unfairly side with the defense’s argument that the police unjustly profiled the defendant.

Should the court sustain the defendant’s objection?

A) No, because the ultimate burden of persuasion regarding racial motivation rests with the opponent of the strike.
B) No, because the prosecutor’s challenges excluded all nonwhite jurors, not just those of the same race as the defendant.
C) Yes, because race is “inextricably bound up” in a case involving allegations of racial profiling.
D) Yes, because the prosecution failed to provide a race-neutral explanation for the peremptory strikes.

A

D) Yes, because the prosecution failed to provide a race-neutral explanation for the peremptory strikes.

The Fourteenth Amendment equal protection clause prohibits striking potential jurors based solely on their race, ethnicity, or sex.

How well did you know this?
1
Not at all
2
3
4
5
Perfectly
32
Q

In a jury trial, an adult defendant was found guilty of first-degree premeditated murder, a capital offense. During the sentencing phase of the trial, the defense presented evidence of mitigating circumstances, while the prosecution presented evidence of aggravating circumstances. The jury rendered an advisory sentence of death without specifying the factual basis for its recommendation. The judge independently found the existence of two aggravating circumstances beyond a reasonable doubt, weighed the aggravating and mitigating circumstances, and, agreeing with the jury’s advisory sentence, imposed the death sentence.

On appeal, the defendant has challenged the imposition of the death sentence as unconstitutional.

How should the appellate court rule?

A) For the defendant, because the judge imposed the death sentence.
B) For the defendant, because the jury did not find a specific aggravating circumstance that justified the imposition of the death penalty.
C) Against the defendant, because the jury and the judge both concurred with imposing the death sentence.
D) Against the defendant, because the judge found two aggravating circumstances beyond a reasonable doubt.

A

B) For the defendant, because the jury did not find a specific aggravating circumstance that justified the imposition of the death penalty.

Due Process: requires that any fact (e.g., aggravating circumstances) that exposes a criminal defendant to a greater punishment than authorized by the jury’s guilty verdict is an element that must be submitted to and decided by the jury.

Imposing the death sentence here was unconstitutional and the court of appeals should rule for the defendant.

How well did you know this?
1
Not at all
2
3
4
5
Perfectly
33
Q

A defendant was charged with both battery and robbery of a victim. The defendant was found guilty of battery but acquitted of the robbery charge. Subsequently, the victim died from the injuries inflicted by the defendant. The defendant has been charged with felony murder of the victim. The defendant has moved for dismissal of this charge on double jeopardy grounds.

How is the court likely to rule on this motion?

A) Grant the motion, because the defendant was convicted of battery.
B) Grant the motion, because the defendant was acquitted of robbery.
C) Deny the motion, because the victim’s death took place after the defendant’s first trial.
D) Deny the motion, because of the collateral-estoppel doctrine.

A

B) Grant the motion, because the defendant was acquitted of robbery.

A felony-murder prosecution predicated upon an underlying felony for which the defendant was acquitted in a previous trial is improper = requires retrying the underlying felony in violation of the double jeopardy clause.

Note: A conviction of a lesser included offense prior to the occurrence of an event necessary to establish a greater offense does not preclude trial of the greater offense. Therefore, the fact that the defendant was convicted of battery is not a basis to grant the motion to dismiss.

How well did you know this?
1
Not at all
2
3
4
5
Perfectly
34
Q

A manager runs a soup kitchen for malnourished children whose parents cannot afford to provide their children with freshly cooked, nutritious meals. So that the soup kitchen can remain financially viable, the manager often accepts free ingredients from a local food bank. One day, the manager received 400 pounds of potatoes from the food bank. He had very few other ingredients on hand that day, so he decided to serve a healthy potato soup for the children. The manager started opening the potato bags and noticed that a majority of the potatoes had green eyes and greenish skin. When he gave the potatoes to his volunteer chef, the chef mentioned that green potatoes are unsafe and potentially toxic, especially when fed to children. The manager brushed her off and said that the green potatoes were just unripe. The chef refused to cook with the green potatoes, so she left the soup kitchen and the manager made the potato soup by himself.

A few weeks later, many of the children who had ingested the potato soup ended up in comas. The doctors were able to trace the cause of the comas back to the toxic green potatoes. The doctors saved all but two of the children, who never came out of the comas and eventually died.

The jurisdiction defines first-degree murder as murder committed willfully with premeditation and deliberation, second-degree murder as all other murder at common law, and voluntary manslaughter as at common law.

The following are listed in descending order of seriousness. What is the most serious offense of which the manager can be properly convicted?

A) First-degree murder.
B) Second-degree murder.
C) Voluntary manslaughter.
D) No form of criminal homicide.

A

B) Second-degree murder.

depraved-heart murder – an unintentional killing that results from reckless indifference to an unjustifiably high risk to human life.

Here, the manager was told that green potatoes are potentially toxic, especially when fed to children. The manager “brushed off” this warning and fed green potatoes to the children anyway. This fact is likely sufficient to hold the manager liable for depraved-heart murder, which is classified as second-degree murder in the jurisdiction

Note: Voluntary manslaughter requires an intentional killing. Since the manager did not intent to kill the children—he believed that the green potatoes were merely unripe—he cannot be convicted of this crime.

How well did you know this?
1
Not at all
2
3
4
5
Perfectly
35
Q

An employee was up for a promotion but was passed over by his boss for a female colleague with more experience. After learning that he had not received the promotion, the employee became angry with his boss and convinced himself that the colleague and the boss, who was married, were involved in a relationship. He thereafter contacted the boss’s wife and convinced her to murder the boss. The employee and the wife stated that they would not harm the colleague, as it might make their involvement too obvious. The employee provided the wife with a gun.

The next day, the wife approached the colleague and the boss in their office parking lot. The wife, who was not an experienced shooter, shot the colleague in the arm, panicked, and then ran off. An onlooker rushed the colleague to the hospital. Although the injury was not life threatening, the colleague contracted an infection during surgery and died the following week. A later investigation revealed that the infection was a result of medical malpractice that occurred during the surgery. The crime was eventually traced back to the employee and the wife, and they were both charged in connection with the colleague’s death.

The employee is most likely to be convicted of which of the following crimes?

A) Murder, attempted murder, and conspiracy to commit murder.
B) Murder and conspiracy to commit murder only.
C) Attempted murder and conspiracy to commit murder only.
D) Murder only.

A

A) Murder, attempted murder, and conspiracy to commit murder.

Pinkerton rule: conspirator is criminally liable for any foreseeable crimes committed by a coconspirator acting in furtherance of the conspiracy.

The conspiracy is completed upon the occurrence of the agreement and (if required) the overt act—the unlawful purpose (to murder the boss) need not be accomplished. Therefore, the employee can be convicted of conspiracy to commit murder.

Here, the colleague would not have died but for the wife’s conduct (actual causation), and the colleague’s death was a foreseeable consequence of that conduct (proximate causation)—notwithstanding the medical malpractice. And since the killing was committed in furtherance of the conspiracy, the wife and the employee—as her coconspirator—are also guilty of murder

How well did you know this?
1
Not at all
2
3
4
5
Perfectly
36
Q

A gardener entered a garden center during regular business hours. He intended to steal some heirloom parsley plants for his garden. He noticed a display of regular parsley plants at the back of the store, but mistakenly believed they were a very valuable type of heirloom parsley. When no one was looking, he took two small parsley plants from the sales display, put them under his sweatshirt, and left the store.

At the register, there was a sign that advertised, “Like parsley? We like it so much that we grew more than we need this season! Today only, we are offering each customer two free parsley plants from our surplus!” The gardener never saw the sign.

Burglary in the jurisdiction is defined as “entering any building unlawfully with the intent to commit a crime.” The jurisdiction follows the common-law definition of larceny. The following crimes are listed in descending order of seriousness.

Which is the most serious crime(s) of which the gardener may be convicted?

A) Burglary and larceny.
B) Burglary only.
C) Larceny only.
D) Attempted larceny only.

A

D) Attempted larceny only.

larceny requires a trespassory taking, so a person cannot commit larceny by taking property freely given by another—even if the person was unaware of this fact.

However, the gardener can be convicted of attempt, which occurs when a person: 1) has the specific intent to commit a crime (here, larceny); 2) acts in furtherance of that crime (by taking and leaving the store with two parsley plants) BUT does not complete it (the plants were free, so no larceny occurred).

How well did you know this?
1
Not at all
2
3
4
5
Perfectly
37
Q

A woman suffered from a debilitating disease, and her husband convinced her that having sexual intercourse with him would cure the disease. The husband knew that his statement was false. Relying on this statement, the woman gave her consent, and the two had sexual intercourse. Later, the woman learned that intercourse could not and did not cure her disease, and she notified the police.

Rape is statutorily defined in this jurisdiction as “sexual intercourse with a female against her will.” The husband was convicted of rape.

If the husband appeals the conviction, how should the appellate court rule on the appeal?

A) Affirm the conviction, because the husband had the specific intent to rape the woman.
B) Affirm the conviction, because the husband obtained the woman’s consent through fraudulent means.
C) Reverse the conviction, because the husband was married to the woman.
D) Reverse the conviction, because the intercourse was not against the woman’s will.

A

D) Reverse the conviction, because the intercourse was not against the woman’s will.

Consent obtained by fraud in factum = not a defense to rape
Consent obtained by fraud in the inducement = valid defense to rape

Here, the husband convinced the woman to have sexual intercourse with him by falsely claiming that it would cure her debilitating disease. Since the woman knew that the act to which she consented was sexual intercourse, her consent was obtained by fraud in the inducement. This type of fraud did not negate the woman’s consent.

How well did you know this?
1
Not at all
2
3
4
5
Perfectly
38
Q

A man spent the afternoon at the beach with his girlfriend. Each of them had consumed a significant quantity of alcohol. The man saw one of the town’s wealthiest residents arrive at the beach, spread out a beach towel, put a large cloth bag on the towel, drop what looked like a wallet into the bag, and run into the ocean. The man recounted his observations regarding the wealthy man’s actions to his girlfriend. She didn’t respond, but she walked over to the bag and opened it. The bag did not contain the wealthy man’s wallet, but it did contain an expensive ring. She took the ring and closed the bag. The wealthy man’s companion, who was just coming onto the beach, observed the girlfriend’s action and alerted a nearby police officer. The girlfriend was arrested. Soon thereafter, the man was arrested as well.

The man is charged with, among other crimes, conspiracy to commit larceny. The applicable jurisdiction has adopted the bilateral approach to conspiracy.

Which of the following is the WEAKEST argument that the man can advance in defense of the conspiracy charge?

A) The bag did not contain a wallet, making it impossible for the man or his girlfriend to have taken it.
B) The girlfriend’s intoxicated state prevented her from forming the intent necessary to commit larceny.
C) The man’s conversation with his girlfriend was inadequate to form a conspiracy agreement.
D) The man’s intoxicated state prevented him from forming the intent necessary to commit larceny.

A

A) The bag did not contain a wallet, making it impossible for the man or his girlfriend to have taken it.

Factual impossibility—i.e., the existence of an unknown condition that makes the unlawful objective impossible to complete—is never a defense to conspiracy.

How well did you know this?
1
Not at all
2
3
4
5
Perfectly
39
Q

A man who worked nights was unable to sleep during the day because of the persistent barking of a neighbor’s dog. Despite repeated requests to the neighbor to address the problem, the barking persisted. The man decided to purchase a pistol to kill the dog. Unfamiliar with firearms, the man bought a pistol but accidentally bought blank ammunition. Standing in his own yard, the man fired several shots at the barking dog that was about 10 feet away in his neighbor’s yard, thinking he was firing real bullets. The dog was unharmed because the blanks were incapable of inflicting harm.

In the applicable jurisdiction, malicious destruction of property is a statutory crime. The highest court of this jurisdiction has ruled that this statutory crime requires the reckless destruction, injury, or defacement of the property of another.

Is the man likely to be found guilty of attempted malicious destruction of property?

A) No, because a reasonable person would have been aware that the blank bullets would not harm the dog.
B) No, because it was impossible for the man to have killed the dog.
C) Yes, because the man acted recklessly in shooting at the dog.
D) Yes, because the man was unaware that the blank bullets would not harm the dog.

A

D) Yes, because the man was unaware that the blank bullets would not harm the dog.

Defendant is guilty of an attempted crime if s/he
1) had the specific intent to commit a crime; and
2) performed an act in furtherance of the crime BUT
3) did not complete it.

Here, the man formed the specific intent to kill the neighbor’s dog. To do so, the man purchased a pistol and fired it at the dog (acts). And though it was factually impossible for the man to kill the dog—because he unknowingly bought blanks—this is no defense to attempt

Note: attempt conviction requires proof that the defendant had the specific intent to commit an offense. Therefore, proof that the man merely acted recklessly in firing at the dog would not suffice

How well did you know this?
1
Not at all
2
3
4
5
Perfectly
40
Q

A police officer believed that a particular house was being used to conduct drug deals. He observed people coming and going at all hours, although he did not see any drugs change hands. One afternoon, the officer saw a person drop a small bag on the porch when he was leaving the house. The person picked the bag up, but the officer thought that perhaps some residue remained. The officer went to the police station, got a trained drug-detection dog, and went to the house. He went up onto the porch with the dog, and the dog alerted him to the presence of drugs.

Based on the dog’s reaction, a search warrant was issued for the house. A subsequent search led to the discovery of various kinds of drugs. The owner of the house was arrested and charged with drug possession and distribution. The owner moved to suppress the drugs found, claiming that the use of the trained drug-detection dog constituted an illegal search.

Should the court grant the owner’s motion to suppress?

A) No, because the officer had probable cause to conduct a search based on his observations of the house.
B) No, because the use of a trained dog to sniff for the presence of drugs does not constitute a search.
C) Yes, because the use of a trained dog to sniff for the presence of drugs is a per se constitutional violation.
D) Yes, because the use of a trained dog to sniff for the presence of drugs on the porch of a house constitutes a search.

A

D) Yes, because the use of a trained dog to sniff for the presence of drugs on the porch of a house constitutes a search.

Police have an implied license to briefly enter a person’s curtilage in the same manner as a private individual. But if police intrude in an unusual manner for an uncommon purpose (e.g., to conduct a canine search), a warrant is generally required.

Here, the search was unreasonable because it was not supported by a warrant or an exception, so the drugs obtained as a result of that search should be suppressed.

Note: The officer may have had probable cause to believe that the house was being used to conduct drug deals. However, absent some exception, the officer was still required to obtain a warrant to conduct a canine search.

How well did you know this?
1
Not at all
2
3
4
5
Perfectly
41
Q

A man and a woman were leaders of a gang and drug dealers. Some rival gang leaders burned their house to the ground. The man and the woman then checked into a motel. The police, who had been watching the couple, thought that they had probably taken any remaining drugs with them to the motel. The police went to the motel’s front desk and asked the desk clerk, an employee of the business, to open the couple’s room. When the desk clerk let the police into the couple’s room, the police found bags of cocaine and heroin on the dresser and the bed. The man and the woman were arrested and charged with drug possession with the intent to distribute. The man has moved to suppress the cocaine and heroin.

Will his motion likely succeed?

A) No, because the desk clerk consented to the search.
B) No, because the police had probable cause to search the motel room.
C) Yes, because a guest has a reasonable expectation of privacy in a motel room.
D) Yes, because the desk clerk was not the manager of the motel.

A

C) Yes, because a guest has a reasonable expectation of privacy in a motel room.

Motel guests have a reasonable expectation of privacy in their rooms, so police generally must obtain a warrant to search the room. Police cannot circumvent the warrant requirement by asking motel managers or employees to consent to a warrantless search of a guest’s room because they lack the authority to do so.

How well did you know this?
1
Not at all
2
3
4
5
Perfectly
42
Q

An officer pulled over the defendant for speeding. When he ran the defendant’s driver’s license, the officer saw that there was an outstanding warrant for the defendant’s arrest based on her failure to pay child support. The officer arrested the defendant and placed her in the backseat of his squad car. The officer then returned to the defendant’s car and saw that her purse was sitting on the passenger seat. The officer searched the purse and found a small amount of marijuana, which the defendant was not entitled to have. The defendant was later charged with drug possession. She has moved to suppress evidence of the marijuana.

Is the defendant likely to succeed in having evidence of the marijuana suppressed?

A) No, because the officer was permitted to conduct a search of containers immediately associated with the defendant, including her purse, incident to a valid arrest.
B) No, because the officer was permitted to conduct a search of the passenger compartment of the defendant’s vehicle incident to a valid arrest.
C) Yes, because the officer did not have a reasonable belief that the purse on the passenger seat of the defendant’s vehicle contained evidence of the offense.
D) Yes, because the officer could not search the defendant’s vehicle without probable cause to believe that the vehicle contained contraband.

A

C) Yes, because the officer did not have a reasonable belief that the purse on the passenger seat of the defendant’s vehicle contained evidence of the offense.

Police may conduct a warrantless search incident to arrest when a person has been lawfully arrested and the search is limited to the person’s body and areas within the person’s immediate reach. The vehicle may also be searched it is reasonable to believe that evidence of the crime may be found therein

Note: The purse was not immediately associated with the defendant when she was arrested and placed in the squad car because it was in her car, beyond her reach. But had the purse been on her person, the officer could have searched it

How well did you know this?
1
Not at all
2
3
4
5
Perfectly
43
Q

Responding to a 911 call reporting an apparent drug deal, a uniformed police officer parked on the street in front of the defendant’s home. As the officer got out of her car, the defendant and his wife were standing on the front porch of their home. The officer overheard the defendant say to his wife, “Go inside and hide the stash. They can’t follow you without a warrant.” The officer detained the defendant and his wife, preventing them from entering their house until backup arrived.

At the defendant’s subsequent trial for drug possession, is his statement constitutionally admissible?

A) No, as a violation of the Fourth Amendment.
B) No, as a violation of the Fifth Amendment.
C) Yes, under an exception to the warrant requirement.
D) Yes, because use of the statement does not violate the defendant’s constitutional rights.

A

D) Yes, because use of the statement does not violate the defendant’s constitutional rights.

There is no Fourth Amendment search if an officer hears a statement from a place where the officer has a lawful right to be, and the Fifth Amendment does not protect voluntary incriminating statements.

How well did you know this?
1
Not at all
2
3
4
5
Perfectly
44
Q

A suspect was arrested for kidnapping a child, read his Miranda rights, and placed in the back of a police car. The suspect immediately requested an attorney. While transporting the suspect to the police station, the two officers had a brief discussion between themselves on the immorality of hurting a child. Despite the fact that their conversation was unlikely to elicit an incriminating response from the suspect, after listening to the officers’ conversation, the suspect admitted that he had kidnapped the child. At trial, the suspect moved to suppress his confession.

Should the court grant the motion to suppress?

A) No, because the officers’ conversation was unlikely to elicit an incriminating response.
B) No, because the officers read the suspect his Miranda rights prior to his statement.
C) Yes, because the suspect had requested an attorney.
D) Yes, because the suspect was in police custody.

A

A) No, because the officers’ conversation was unlikely to elicit an incriminating response.

All interrogation must cease if a suspect invokes the Fifth Amendment right to counsel. But if the suspect then makes a volunteered statement not prompted by police interrogation, then that statement can be used against the suspect at trial.

Note: Although the officers read the suspect his Miranda rights prior to the statement, they could not interrogate the suspect after he invoked those rights by requesting an attorney. So had the officers’ conversation been likely to elicit an incriminating response, the suspect’s statement would not have been admissible.

How well did you know this?
1
Not at all
2
3
4
5
Perfectly
45
Q

An undercover officer infiltrated a gang by posing as a gang member. While working undercover, the officer heard that a member of the gang had shot and killed one of the leaders of a rival gang. The undercover officer coordinated with the police department’s gang unit to conduct a sweep whereby each gang member, including the undercover officer, would be taken into custody on minor charges. The officer made sure that he and the gang member suspected of the murder were placed in the same cell. While in the cell, the officer brought up the shooting, indicating that he was very impressed by whoever had the courage to kill the rival gang leader. The gang member, taking the bait, bragged that he had shot the rival gang leader but told the officer to keep the information secret. The gang member was arrested for the murder and the prosecution sought to introduce the statement made to the undercover officer.

Is the statement likely to be admitted?

A) No, because the gang member was questioned by a police officer without receiving Miranda warnings.
B) No, because the officer deceived the gang member as to the officer’s true identity.
C) Yes, because the gang member did not know that the undercover officer was a police officer.
D) Yes, because the gang member was not subjected to interrogation.

A

C) Yes, because the gang member did not know that the undercover officer was a police officer.

Miranda warnings are not required when a suspect who is subjected to a custodial interrogation is not aware that the interrogator is a police officer—i.e., when the officer is undercover.

Here, the gang member was subjected to interrogation while he was in custody (jail cell) because the officer knew or should have known that his words were likely to elicit an incriminating response. But since the gang member did not know that he was talking to a police officer, Miranda warnings were not required

How well did you know this?
1
Not at all
2
3
4
5
Perfectly
46
Q

Police arrested an 18-year-old defendant at her home that she shared with her parents as an accomplice to a robbery. Before leaving the home, the police asked the defendant whether she was present during the robbery. She indicated in the affirmative. As the police were leaving the home with the defendant, the defendant’s mother returned. The mother inquired about the defendant’s arrest and urged the defendant to “come clean to the police.” The defendant did not respond, and the police took her to the police station.

At the station, after the defendant was given Miranda warnings and signed a Miranda waiver, she was questioned in a noncoercive manner by police. Feeling compelled to follow her mother’s advice, the defendant confessed to her involvement in the robbery. The prosecution sought to introduce the defendant’s confession at trial over the objection of her attorney.

Will the court likely sustain the objection?

A) No, because the defendant waived her Fifth Amendment right to remain silent.
B) No, because the defendant did not have a Fifth Amendment right to remain silent once she was arrested.
C) Yes, because the defendant was not specifically told before she confessed to the robbery that her prior incriminating statement could not be used against her.
D) Yes, because the confession was not voluntary as it was given under the mother’s compulsion.

A

A) No, because the defendant waived her Fifth Amendment right to remain silent.

A Miranda violation does not automatically require the suppression of the defendant’s later confession made after the receipt of Miranda warnings. Instead, admissibility turns on whether the later confession was voluntary based on the totality of the circumstances.

How well did you know this?
1
Not at all
2
3
4
5
Perfectly
47
Q

A defendant agreed to help a friend with a burglary by transporting the friend to the scene of the burglary, keeping watch outside the residence while the friend committed the burglary, and then driving the friend away from the scene. The friend believed that no one would be home at the time of the burglary, so the friend told the defendant that he would be unarmed. The friend lied, and instead carried a gun on his person into the residence. After waiting for several minutes, the defendant got cold feet and drove away from the residence, abandoning his friend. Shortly thereafter, the friend encountered an occupant inside the residence. Panicking, the friend shot and killed the occupant.

The defendant has been charged with felony murder in a jurisdiction that permits capital punishment by lethal injection for felony murder.

Can the death penalty be imposed on the defendant?

A) No, because capital punishment cannot constitutionally be imposed for felony murder.
B) No, because the defendant did not kill, attempt to kill, or intend to kill the occupant.
C) Yes, because death by lethal injection does not constitute cruel and unusual punishment.
D) Yes, because the defendant is guilty of felony murder.

A

B) No, because the defendant did not kill, attempt to kill, or intend to kill the occupant.

An accomplice to felony murder who did not kill, attempt to kill, or intend to kill cannot be sentenced to the death unless the accomplice (1) significantly participated in the commission of the underlying felony and
(2) acted with reckless indifference to human life.

How well did you know this?
1
Not at all
2
3
4
5
Perfectly
48
Q

A defendant was tried for felony murder. The jury was unable to reach a unanimous verdict. As a result, the judge declared a mistrial over the defendant’s objection. Subsequently, the state refiled the charges against the defendant.

Is the second prosecution a violation of the defendant’s constitutional protection from double jeopardy?

A) No, because the first trial resulted in a mistrial due to a hung jury.
B) Yes, because the defendant did not consent to the mistrial.
C) Yes, because the double jeopardy clause protects against a second prosecution for the same offense.
D) Yes, because jeopardy attached in the first trial when the jury was impaneled and sworn in.

A

A) No, because the first trial resulted in a mistrial due to a hung jury.

The double jeopardy clause does NOT bar a second prosecution for the same offense when

1) a mistrial is declared at the defendant’s request
2) with the defendant’s consent, or 3) due to manifest necessity (e.g., a hung jury).

How well did you know this?
1
Not at all
2
3
4
5
Perfectly
49
Q

A certified public accountant defrauded a client of money entrusted to the accountant by the client. A state prosecutor charged the accountant with fraud. In a trial without a jury, the accountant was tried and found guilty of fraud and sentenced to imprisonment for one year. Subsequently, the state accountancy board initiated an administrative proceeding against the accountant to revoke the accountant’s license based on his fraudulent conduct. The accountant, contending that the proceeding violates his constitutional protection against double jeopardy, has filed a court petition seeking to quash the proceeding. All of the above actions took place within a single state.

Should the court rule in favor of the accountant?

A) No, because double-jeopardy protections do not apply to administrative proceedings regarding conduct for which a person has already been subjected to criminal punishment.
B) No, because the constitutional protection against double jeopardy does not apply unless the prior criminal punishment was imposed after a jury trial.
C) Yes, because the accountant is being subjected to punishment for conduct for which he has already been punished by imprisonment.
D) Yes, because the fraud trial and the license revocation proceeding were actions brought by the same state.

A

A) No, because double-jeopardy protections do not apply to administrative proceedings regarding conduct for which a person has already been subjected to criminal punishment.

Double jeopardy: does not apply to admin proceedings

How well did you know this?
1
Not at all
2
3
4
5
Perfectly
50
Q

A professional musician’s instrument was stolen from her while she was on tour. An attorney erroneously advised the musician that, if she saw the instrument, she would be legally entitled to use force to regain possession of it in any circumstance. The musician believed the attorney, but in truth, the privilege to use force to recapture property was only available to a person who witnessed the taking of the property and used force immediately thereafter.

Several months later, the musician saw a performer with her instrument at a concert. He had purchased the instrument from the thief without knowing that it had been stolen. When the performer refused to hand over the instrument to the musician, she tried to take the instrument. In the ensuing struggle, the instrument was destroyed.

A statute makes it a crime for a person to use force to recapture property, knowing she has no legal privilege to do so.

If the musician is charged and prosecuted under this statute, can she properly be convicted?

A) No, because she acted on the advice of her attorney.
B) No, because she lacked the necessary mental state.
C) Yes, because reliance on the erroneous advice of an attorney is never a defense.
D) Yes, because she had no privilege to use force to take the instrument.

A

B) No, because she lacked the necessary mental state.

A mistake of law based on erroneous legal advice is generally no defense to criminal liability. However, reliance on such advice may serve as a defense when the mistake negates the required mental state.

How well did you know this?
1
Not at all
2
3
4
5
Perfectly
51
Q

A state statute provides: “Criminal solicitation of a minor consists of soliciting a person who is under 17 years of age to engage in certain enumerated acts, including prostitution.” The jurisdiction aligns its levels of culpability with the definitions provided by the MPC and follows the MPC’s guidance if the requisite mens rea is not stated.

Evidence introduced at a criminal trial showed that the defendant approached a 16-year-old student outside of a high school and solicited the student to engage in prostitution. The defendant testified truthfully that he had had no idea how old the student was and had no intent whatsoever to solicit a minor.

Could the defendant properly be convicted of criminal solicitation of a minor?

A) No, because the evidence showed that the defendant did not knowingly solicit a person under the age of 17.
B) No, because there was no evidence of a substantial risk that the student was under the age of 17.
C) Yes, because the defendant consciously disregarded the risk that the victim was under the age of 17.
D) Yes, because there was clear evidence that the victim was under the age of 17, and the statute is designed to protect minors.

A

C) Yes, because the defendant consciously disregarded the risk that the victim was under the age of 17.

When a statute does not specify the requisite mental state, the minimum required mental state according to the MPC is recklessness—i.e., the conscious disregard of a substantial and unjustified risk that:

the material element exists or
the material element will result from the defendant’s conduct.

52
Q

A sculptor and a painter wanted some art supplies. They were both penniless, so they decided to rob an art supply store. The sculptor gave the painter one of his sharpest knives to use as a weapon. The sculptor remained in his studio while the painter went three miles away to the art supply store, showed the clerk the knife, told him to hand over all the best oil paints and clay, and took the supplies back to his studio. The security camera in the store recorded the entire interaction. The next day, the police arrested the painter for robbery. He told them that the sculptor had given him the knife, and they arrested the sculptor for robbery as well.

At trial, the painter was acquitted of robbery on a technicality. Surprised and frustrated, the prosecutor worked even harder to convict the sculptor.

The jurisdiction follows the majority approach to accomplice liability.

Can the sculptor be convicted of robbery?

A) No, because the painter was acquitted of the robbery charge.
B) No, because the sculptor did not go to the art supply store.
C) Yes, because the painter identified the sculptor as an accomplice.
D) Yes, because the sculptor gave the knife to the painter.

A

D) Yes, because the sculptor gave the knife to the painter.

An accomplice need not participate in or be present at the crime to be held liable and, under the modern majority approach, may be convicted even if the principal is not.

53
Q

A man asked a coworker to lend him something he could use to break into their manager’s house when he was not home in order to steal the manager’s valuable record collection. In fact, unbeknownst to the coworker, the man had no intention of stealing the records, but actually intended to break into the house when the manager was home in order to assault the manager. The next day, the coworker brought a lockpick to work and handed it to the man. The man took the lockpick but, upon arriving at the manager’s house that night, found that the door had been left unlocked. The man entered the manager’s house and assaulted the manager. As he prepared to leave the manager’s home, the man noticed the records and took them. He sold the records to a collector later that week.

The coworker has been charged as an accomplice to burglary, larceny, and assault.

Of which crime(s), if any, is the coworker guilty?

A) Burglary, larceny, and assault.
B) Burglary and larceny only, because she did not intend to assist in the assault.
C) Larceny only, because her lockpick was not used to accomplish the breaking.
D) No crime, because she did not provide any substantial aid or assistance with any crime.

A

B) Burglary and larceny only, because she did not intend to assist in the assault.

Accomplice liability is imposed when a person provides even slight aid or encouragement to the principal with the specific intent that the encouraged crime be completed—even when that assistance is not ultimately necessary to complete the crime.

Assault is not the natural and probable consequence of a burglary of an unoccupied home, so the coworker is not guilty of this crime.

54
Q

An accountant bought a small amount of cocaine from a dealer. As she walked away, she had second thoughts. She turned around and walked back toward the dealer in order to negotiate a return of the cocaine. An undercover police officer observed these events, and before the accountant could return the cocaine, the officer arrested both her and the dealer. The dealer was charged pursuant to a drug-distribution statute under which the distributor of illegal drugs can be held criminally liable, and the accountant was charged with distributing drugs as an accomplice. The dealer was tried first but was not convicted. The accountant is now being tried.

The jurisdiction follows the majority rules for accomplice liability.

Should the court find the accountant guilty as an accomplice?

A) No, because the drug-distribution statute does not impose liability on the purchaser.
B) No, because the principal was never convicted of the crime.
C) Yes, because her uncommunicated decision to withdraw from the sale was ineffective.
D) Yes, because she intended to purchase the cocaine.

A

A) No, because the drug-distribution statute does not impose liability on the purchaser

Here, the accountant was charged with distributing drugs as an accomplice. The accountant aided the dealer in the distribution of cocaine and specifically intended that the dealer sell her cocaine. But since drug-distribution statutes do not impose liability on the purchaser, the accountant acted as the nonliable participant to the distribution.

Note: The accountant would have been guilty as an accomplice had she encouraged the dealer to sell drugs to a third party.

Note: this conduct may make the accountant guilty of a separate crime.

55
Q

A man accidentally crashed his truck into his neighbor’s vintage car. The neighbor, who had been planting rose bushes in front of his house, witnessed the accident. The neighbor instinctively grabbed his shovel and ran toward the man as he was exiting his truck. The neighbor struck the man on the left side of his head, injuring the man and causing his death a week later.

The following crimes are listed in descending order of seriousness in a jurisdiction that follows common-law principles.

What is the most serious crime of which the neighbor could be convicted?

A) Murder.
B) Voluntary manslaughter.
C) Attempted murder.
D) Assault.

A

A) Murder.

A defendant is guilty of common-law murder when a jury finds that the defendant unlawfully killed another with malice aforethought—e.g., intent to kill or inflict serious bodily harm.

Note: Accidental damage to property is highly unlikely to constitute adequate provocation. But even if it were, a jury could find the neighbor guilty of the more serious crime of murder.

56
Q

A woman was walking down a city street when the defendant grabbed her and told her to hand over her wallet. The defendant did not have a weapon, nor did he claim to have one. The woman broke away from the defendant using the techniques she had learned in a self-defense course, and she began running. Fearing apprehension, the defendant ran in the opposite direction. After sprinting two city blocks, the woman realized that the defendant was no longer nearby. However, still flustered from the incident, she wandered into the street where she was hit by a taxi that was driving 10 miles per hour over the speed limit. The woman later died from the injuries she sustained.

The defendant was charged with felony murder in a jurisdiction that has adopted the proximate-cause theory.

What is the defendant’s best argument that he is not guilty of felony murder?

A) The death was not a natural and probable consequence of the robbery.
B) The defendant did not engage in an inherently dangerous felony, because he did not have a weapon.
C) The defendant did not intend to cause the woman physical harm.
D) The taxi driver could not be considered the defendant’s agent.

A

A) The death was not a natural and probable consequence of the robbery.

Under the proximate-cause theory of liability, a felony-murder defendant is responsible for any death that is a natural and probable consequence of the felony.

Note: under the agency theory of liability, the defendant is only responsible for deaths caused by the defendant’s agent.

The taxi driver was not the defendant’s agent. But this would only be relevant in a jurisdiction that had adopted the agency theory of felony-murder liability (not seen here).

57
Q

A husband and wife took their 12-year-old son to a political rally in an auditorium to hear a controversial United States senator speak. The speaker was late, and the wife stepped outside the auditorium to smoke a cigarette. While there, she saw a man placing what she believed to be a bomb against the back wall of the auditorium. She went back inside and told her husband what she had seen. Without alerting anyone, they took their son and left. Some 20 minutes later, the bomb exploded, killing 8 persons and injuring 50. In the jurisdiction, murder in the first degree is defined as an intentional homicide committed with premeditation and deliberation; murder in the second degree is defined as all other murder at common law; and manslaughter is defined as either a homicide in the heat of passion arising from adequate provocation or a homicide caused by gross negligence or reckless indifference to consequence.

As to the deaths of the eight persons, what crime, if any, did the wife commit?

(A) Manslaughter
(B) Murder in the first degree.
(C) Murder in the second degree.
(D) No crime.

A

(D) No crime

The wife did not have a legal duty, enforceable by the criminal laws, to warn others about the bomb.

58
Q

A woman decided to steal a necklace that belonged to her neighbor. She knew where the neighbor kept the necklace because she had been in the neighbor’s house on many occasions when the neighbor had taken off the necklace and put it away in a jewelry box in the bathroom. One night, the woman went to the neighbor’s house. The neighbor was away, and the house was dark. The woman opened the bathroom window, saw the jewelry box on the counter, and started to climb inside. As her leg cleared the window sill, the neighbor’s dog began to bark loudly. Terrified, the woman jumped back outside and fled.
The crimes below are listed in descending order of seriousness.

What is the most serious crime, if any, committed by the woman?

(A) Burglary.
(B) Attempted burglary.
(C) Attempted larceny.
(D) No crime.

A

(A) Burglary.

The woman unlawfully entered the neighbor’s house at night with intent to commit a felony (larceny). The woman’s actions constituted the requisite entry of the neighbor’s house.

Note: The woman’s action proceeded beyond the point of attempted burglary. The woman is guilty of burglary because she unlawfully entered the neighbor’s house at night with intent to commit a felony (larceny). The woman’s actions constituted the requisite entry of the neighbor’s house.`

59
Q

A customer asked to see an expensive watch in a jewelry store. In conversation with the store clerk, the customer falsely claimed to be the son of the mayor. When handed the watch, the customer asked if he could put it on, walk around a bit so he could see how it felt on his wrist, and then briefly step outside to observe it in natural light. The clerk agreed, saying, “I know I can trust someone like you with the merchandise.” The customer walked out of the store wearing the watch and did not return. A week later, the clerk was at a gathering when she spotted the customer wearing the watch. She told him that he must either pay for the watch or give it back. He hissed, “You’ll be sorry if you mess with me.” Intimidated, the clerk backed off.
The following list of crimes is in descending order of seriousness.

What is the most serious crime the customer committed?

(A) Robbery.
(B) Larceny.
(C) False pretenses.
(D) Embezzlement.

A

(B) Larceny.
The customer committed a trespassory taking and carrying away of another’s property with the intent to steal it. He obtained possession of, but not title to, the watch by lying about a present fact.

Note: The crime was larceny, not false pretenses, because the customer obtained possession of, but not title to, the watch.

60
Q

After a liquor store was robbed, the police received an anonymous telephone call naming a store employee as the perpetrator of the robbery. Honestly believing that their actions were permitted by the U.S. Constitution, the police talked one of the employee’s neighbors into going to the employee’s home with a hidden tape recorder to engage him in a conversation about the crime. During the conversation, the employee admitted having committed the robbery.

The employee was charged with the robbery in state court. He has moved to suppress the recording on the grounds that the method of obtaining it violated both his federal and his state constitutional rights.
Assume that a clear precedent from the state’s highest court would result in a finding that the conduct of the police in making the recording violated the employee’s rights under the state constitution, and that excluding the recording is the proper remedy.

Should the court grant the employee’s motion?

A) No, because the employee’s federal constitutional rights were not violated, and this circumstance overrides any state constitutional provisions.
(B) No, because the police were acting in the good-faith belief that the federal Constitution permitted their actions.
(C) Yes, because the making of the recording violated the state constitution.
(D) Yes, because use of the recording would violate the neighbor’s federal constitutional rights.

A

(C) Yes, because the making of the recording violated the state constitution.

A state may grant broader rights under its own constitution than are granted by the federal Constitution. Here, the state has a clear precedent establishing that the recording violated the employee’s state constitutional rights and that it should be excluded from evidence as a remedy. The court should apply this precedent to grant the employee’s motion.

Note: It is irrelevant what the police thought about the propriety of their actions under the federal Constitution where the state has granted broader rights under its own constitution

61
Q

A police officer obtained a valid warrant to arrest a woman for misdemeanor theft. Having probable cause to believe that the woman was spending the afternoon at a friend’s house, the officer went to the friend’s house to serve the warrant. No one responded to the officer’s knocking or to his identification of himself as a police officer. The officer, finding the door unlocked, opened the door and entered the house. Once in the house, the officer found the woman hiding in a bedroom closet. The woman was properly charged with misdemeanor theft. She sought dismissal of the charge against her due the manner of her arrest.

Should the court dismiss the charge against the woman?

A) No, because the officer had probable cause to believe that the woman was in the friend’s house.
B) No, because the woman was properly charged with misdemeanor theft.
C) Yes, because the officer could not arrest the woman for a misdemeanor unless the misdemeanor occurred in the officer’s presence.
D) Yes, because the officer’s arrest of the woman at her friend’s house was illegal.

A

B) No, because the woman was properly charged with misdemeanor theft.

Police may enter a third party’s home to legally execute an arrest warrant only when they have
(1) a search warrant;
(2) exigent circumstances; or
(3) the third party’s consent.

BUT an illegal arrest does not bar subsequent prosecution of the arrestee, so long as the arrestee was properly charged.

Here, the arrest of the woman in her friend’s home was illegal because the police did not have a warrant to search the friend’s home, exigent circumstances, or the friend’s consent to enter the home. However, that illegal arrest will not prevent the subsequent prosecution of the woman for misdemeanor theft since she was properly charged

62
Q

A police officer’s wife often worked late with her business partner. The officer suspected that they were having an affair and decided to confront the partner. After putting on his patrol uniform and badge, the officer went to the partner’s home and kicked open the front door. He found the partner in his kitchen putting cocaine into little bags. The officer immediately arrested the partner for possession of cocaine and informed him of his Miranda rights. The officer confronted the partner about the affair, and the partner confessed that he and the officer’s wife spent late nights together selling cocaine.

At the partner’s subsequent trial for possession and distribution of cocaine, the prosecution conceded that the partner’s arrest was unlawful. The partner then moved to suppress evidence of his confession, arguing that it was involuntary.

What is the strongest argument in support of the suppression of the confession?

A) The confession was involuntary because the partner did not waive his Miranda rights.
B) The confession was too closely tied to the unlawful arrest.
C) The officer did not have probable cause to detain the partner.
D) The partner made the confession after the officer unlawfully arrested him.

A

B) The confession was too closely tied to the unlawful arrest.

An incriminating statement taken after an unlawful arrest is admissible if the court determines that the connection between the arrest and the statement is so attenuated that the statement is considered voluntary.

Note: An uncoerced confession made after an unlawful arrest will not automatically be suppressed. The court must still determine if the confession is too closely connected to the arrest to be considered voluntary.

63
Q

Police executed a valid warrant to search for heroin in the defendant’s residence. Finding the defendant at home, the police detained him in handcuffs for the duration of the search. The police found a small amount of heroin in the defendant’s bedroom during the search. Upon the completion of the search, the police arrested the defendant for possession of heroin. The police then searched the defendant’s person and found a larger quantity of cocaine. At his trial for possession of cocaine, the defendant sought to suppress the cocaine as having been unconstitutionally seized from his person.

Should the court suppress the cocaine?

A) No, because the cocaine was found pursuant to a search incident to a valid arrest.
B) No, because the police were authorized to seize the cocaine by a search warrant.
C) Yes, because the handcuffing of the defendant constituted a per se unreasonable seizure.
D) Yes, because the police lacked probable cause to arrest the defendant for possession of cocaine at the time of his detention.

A

A) No, because the cocaine was found pursuant to a search incident to a valid arrest.

The “search incident to a lawful arrest” exception allows police to conduct a warrantless search of
(1) a person who has been lawfully arrested and
(2) the areas within that person’s immediate reach.

Note: probable cause developed once the police found cocaine on the defendant while searching him incident to a lawful arrest.

64
Q

Police in a rural community obtained an anonymous tip that a resident of the community was producing opium from poppies grown in flowerbeds immediately behind his house. Since police could not access or view the beds without trespassing on the resident’s property, the police contacted a local crop duster who used a helicopter in his business. At the police’s behest, the crop duster took an officer in his helicopter and flew over the residence at a height of 500 feet, confirming that illegal poppies were growing in the flowerbeds behind the house. The police then obtained a warrant to search the area directly behind the house and seized the illegal plants.

If the resident moves to suppress this evidence as a violation of his Fourth Amendment rights, how is the court likely to rule?

A) Deny the motion, because a civilian flew the helicopter over the residence.
B) Deny the motion, because the inspection did not violate a reasonable expectation of privacy.
C) Grant the motion, because an anonymous tip cannot form the basis for probable cause to obtain a warrant.
D) Grant the motion, because the poppy plants were located within the curtilage of the residence.

A

B) Deny the motion, because the inspection did not violate a reasonable expectation of privacy.

Police flyovers are not Fourth Amendment searches
(1) aerial observations are not physical intrusions and
(2) a person has no reasonable expectation of privacy in items and areas exposed to public view.

Note: though the flower beds behind the house were part of the curtilage, the officer’s flight did not physically intrude upon the area

65
Q

A brutal murder occurred at a roadside rest area. Unable to determine the identity of the killer, the police set up a checkpoint on the highway near the rest area several days later at approximately the same time of night as the murder occurred. The purpose of the checkpoint was to briefly ask travelers for help in solving the murder. The police stopped every car and gave the drivers a flyer with information about the murder. The officer then asked the drivers if they had any relevant information they wanted to share with police. If the drivers responded negatively, they were allowed to continue on their way. If a driver responded affirmatively, the car was directed to the shoulder of the road to talk with another officer.

A driver who was stopped in this manner was smoking marijuana. The police officer, upon smelling the marijuana, directed the driver to the side of the road where he was arrested. At the driver’s trial for driving under the influence, he moves to exclude the evidence of the marijuana from the trial on the ground that the stop violated his constitutional rights.

Should the court grant the driver’s motion?

A) No, because the police’s actions did not constitute a seizure for purposes of the Fourth Amendment because the driver was not subject to an interrogation prior to his arrest.
B) No, because the police had an information-seeking purpose in stopping the driver.
C) Yes, because the arrest of the driver for a drug-related offense occurred as a result of a checkpoint stop.
D) Yes, because the stop was not appropriately tailored to protect the driver’s Fourth Amendment rights.

A

B) No, because the police had an information-seeking purpose in stopping the driver.

SCOTUS has held that a checkpoint at which motorists were stopped so that officers could ask for information about a specific crime committed on that roadway was reasonable because:

1) primary law enforcement purpose was to elicit evidence to help police apprehend individuals other than the vehicle’s occupants

2) the stop significantly advanced a public concern

3) the police tailored the checkpoint to fit important criminal investigatory needs and to minimally interfere with Fourth Amendment rights

Note: though a checkpoint for the general purpose of uncovering drug-related crimes is unconstitutional, the checkpoint here served a special law enforcement need.

66
Q

The police obtained information from a reliable, known informant that the owner of a small tavern was dealing heroin out of the tavern and was expecting a shipment the following day. Based solely on this information, the police obtained a warrant to search the tavern and the owner’s person on the following day after delivery of the shipment. The warrant provided for the police to seize narcotics and related contraband, as well as other fruits, instrumentalities, and evidence of the crime at the time unknown.

When the police arrived at the tavern, they found the owner behind the bar and a few patrons scattered throughout. The defendant, whom the police did not recognize, was sitting in a booth near the bar. One officer searched the patrons. The search of the defendant uncovered a handgun with an obliterated serial number, which the officer seized. It was later revealed that the handgun was stolen, that the defendant was a convicted felon, and that the gun was possessed illegally. The defendant was charged with crimes related to possession of the handgun. The defendant has moved to suppress evidence of the handgun, arguing that it was illegally seized.

What is the defendant’s best argument that the handgun should be suppressed?

A) The police did not have independent justification to search the defendant.
B) The police did not seek judicial authorization to search the patrons of the tavern.
C) The search warrant constituted an impermissible anticipatory warrant.
D) The search warrant was not based on probable cause.

A

A) The police did not have independent justification to search the defendant.

When executing a warrant, police may not lawfully search a person who is not named in the warrant without independent justification.

Such justification may come from
(1) reasonable suspicion that the person is armed or
(2) probable cause to believe that the person committed, is committing, or is about to commit a crime.

Note: The police did not seek judicial authorization to search the tavern’s patrons. However, no such authorization would have been required if police had developed independent justification to conduct those searches.

67
Q

Two police officers detained a woman at the scene of the murder of a gas station attendant. They suspected the woman of committing the crime and brought her to the police station, over her strong objection, for questioning. After one question, asking why she had been present at the crime scene, the woman suddenly burst into tears and shouted out, “That liar cheated on me and I wanted him dead.” One of the officers immediately interrupted the woman, realizing that they had forgotten to give her a Miranda warning at the scene of the crime because of the difficult time they had in detaining her. The officer then gave the woman a full Miranda warning and asked her if she wanted to continue with her statement. The woman then refused to talk any further and demanded to see her attorney. At trial, the woman took the stand in her own defense and testified that she did not know the gas station attendant and had no reason to kill him.

If the court allows the prosecutor to use the statement the woman made to the officers during her interrogation, how may it be used?

A) As direct evidence of the murder.
B) For impeachment purposes only.
C) Both as direct evidence of the murder and for impeachment purposes.
D) None of the above.

A

B) For impeachment purposes only.

Statements taking in violation of Miranda cannot be used directly in deciding ultimate issues of guilt or innocence.

BUT such a statement can be used for the limited purpose of impeaching a criminal defendant’s inconsistent testimony if the statement was voluntary and trustworthy.

68
Q

Police had probable cause to believe that a man had murdered his former business partner, so they obtained a warrant to arrest him. Early the following morning, a team of officers went to the man’s home and, without announcing their presence, kicked open the front door. The officers entered the home with guns drawn and located the man in the kitchen eating breakfast. Startled at the sight of the officer’s guns, the man immediately put his hands in the air. An officer then asked the man if he had killed his former business partner. The man reluctantly replied that he had. The man was placed under arrest and subsequently charged with murder.

If the court suppresses the use of this statement as evidence, what is the most likely reason why?

A) The police did not announce their presence prior to entering the man’s home.
B) The police did not give the man the required Miranda warnings.
C) The police did not have a search warrant authorizing entry into the house.
D) The statement was coerced because it was obtained while the man was at gunpoint.

A

B) The police did not give the man the required Miranda warnings.

Note: Here, the statement was obtained while the man was at gunpoint, but there is not indication that the statement was solely the product of coercion.

Totality of circumstances: level of police coercion, length of interrogation, suspect’s characteristics, Miranda warnings

69
Q

A woman was suspected of murder in the shooting death of her husband and was taken into custody. Two officers brought her to an interview room and told her that another officer would arrive to conduct her interrogation shortly. They did not read the woman her Miranda rights, but instead waited quietly in the room with her for the other officer to arrive. After 30 minutes of waiting, the woman became extremely agitated and blurted out, “I did it! I killed him, and I threw the gun into the river!” The police dragged the river and recovered a gun, and ballistics tests confirmed that the gun was the murder weapon. At trial, the woman’s attorney moved to suppress introduction of both the confession and the gun as evidence.

How should the court rule?

A) Suppress both the confession and the gun.
B) Suppress only the confession.
C) Suppress only the gun.
D) Suppress neither the confession nor the gun.

A

D) Suppress neither the confession nor the gun.

Volunteered statements are not the product of interrogation and are therefore admissible even if the suspect had not been Mirandized.

70
Q

A defendant was charged with capital murder, and the state sought the death penalty. During voir dire, the prosecution challenged nine prospective jurors for cause when the prospective jurors indicated that they were opposed to the death penalty and could never vote to impose it. The defendant has objected to the prosecutor’s challenges.

How should the court rule on the defendant’s objection?

A) Overrule the objection, because a party may base a challenge for cause on any non-race-related reason.
B) Overrule the objection, because jurors hearing a death penalty case must be able to impose the death penalty.
C) Sustain the objection, because the defendant is entitled to be tried by an impartial jury chosen from a fair cross section of the community.
D) Sustain the objection, because the prospective jurors’ views would not prevent them from impartially deciding the issue of the defendant’s guilt.

A

B) Overrule the objection, because jurors hearing a death penalty case must be able to impose the death penalty.

A potential juror whose views would substantially impair his/her ability to impartially decide the case can be challenged for cause.

Challenge for cause: based on a specific reason that would disqualify the prospective juror from service

Preemptory challenge: remove a prospective juror for any reason other than race, ethnicity, or gender.

71
Q

A defendant has been charged with embezzlement for allegedly failing to return a car the defendant rented from a vehicle rental company. At trial, the prosecutor requested that the judge instruct the jury on the following presumption: A person who willfully and intentionally fails to return a rented vehicle within five days after the rental agreement has expired shall be presumed to have intended to defraud the owner of the vehicle.

Is the instruction proper?

A) No, because mandatory presumptions are not allowed against a criminal defendant on an element of the charged crime.
B) No, because the fact that a rented vehicle was not returned within five days of the expiration of the rental agreement does not necessarily lead to a conclusion that there was an intent to defraud.
C) Yes, because it expresses a rational conclusion that the jury should be required to accept.
D) Yes, because the defendant has a chance to rebut the presumption by offering evidence that he had no intent to defraud the owner of the vehicle.

A

A) No, because mandatory presumptions are not allowed against a criminal defendant on an element of the charged crime.

Due process prohibits the use of mandatory presumptions—i.e., conclusions that must be drawn from basic facts—against a criminal defendant regarding an element of the charged crime.

72
Q

A defendant was tried for burglary of a jewelry store. At trial, a witness for the prosecution testified that he saw the defendant loitering outside the jewelry store on the night of the burglary. A defense witness, the defendant’s employer, then testified that the defendant was working the night shift across town on the night of the burglary. On cross-examination, the prosecutor confronted the employer with a statement she had given the police in which she said that she could not remember who was working on the night of the burglary. The employer acknowledged the statement but testified that she was mistaken when talking to the police.

As to the defendant’s alibi, which of the following instructions would be proper?

A) If the defendant’s evidence has given you reasonable doubt as to whether he was the burglar, you must find him not guilty.
B) If you have reasonable doubt as to whether the defendant was present at the jewelry store on the night of the burglary, you must find him not guilty.
C) While the prosecution must prove all elements of the crime beyond a reasonable doubt, the defendant must establish his alibi by a preponderance of the evidence.
D) You must decide whether the defendant has produced sufficient evidence to raise the issue of alibi before you may consider it on its merits.

A

B) If you have reasonable doubt as to whether the defendant was present at the jewelry store on the night of the burglary, you must find him not guilty.

Due process requires that the prosecution prove every element of a criminal offense beyond a reasonable doubt to convict a defendant—even when the defendant asserts a defense that negates an element of the crime.

Note: when the defendant asserts a defense (e.g., alibi) that negates an element of the crime, the burden of proof must remain on the prosecution to prove that element. In other words, the burden may not be shifted to the defendant to disprove the element

73
Q

A defendant was charged with the misdemeanor offense of indecent exposure after it was alleged that he exposed himself at a public park. Upon the advice of counsel, the defendant waived a jury trial and pleaded guilty. Thereafter, the defendant was sentenced to 120-days’ confinement in the county jail.

A month later, after discovering that a child was present when the defendant exposed himself, the prosecutor sought and obtained an indictment against the defendant for the felony offense of indecency with a child by exposure. The state’s highest court has ruled that indecent exposure is a lesser included offense of indecency with a child by exposure. The defendant has moved to dismiss the charge on double-jeopardy grounds.

Should the court rule in favor of the defendant?

A) No, because the defendant pleaded guilty to the lesser included offense.
B) No, because the discovery that a child was present constitutes the occurrence of a necessary subsequent event.
C) Yes, because the defendant has yet to complete his sentence for the lesser included offense.
D) Yes, because the defendant pleaded guilty to the lesser included offense.

A

D) Yes, because the defendant pleaded guilty to the lesser included offense.

Conviction of a lesser included offense stems from a guilty plea –> the double jeopardy clause bars a subsequent prosecution for the greater offense unless (1) an event necessary to establish the greater offense occurred after the plea was entered or
2) the greater offense was charged before the plea was entered.

74
Q

While on patrol, a police officer saw a known drug dealer drive by alone in his car. The officer suspected that the dealer was in possession of drugs, but he lacked probable cause to stop the dealer on that basis. The officer noticed that the windows of the car were tinted in excess of what is permitted by state law. The officer pulled the dealer over for violation of the window-tinting law. While issuing the ticket to the dealer, the officer smelled marijuana smoke emanating from the vehicle. The officer ordered the driver out of the car and searched it, finding not marijuana, but a bag of cocaine on the floor of the trunk.

At trial for drug possession, the dealer sought to suppress the cocaine. If the court grants this suppression motion, which of the following is most likely justification for doing so?

A) The stop for violation of the window-tinting law was merely a pretext for determining whether the dealer was in possession of drugs.
B) The officer was searching for marijuana, not cocaine.
C) The officer lacked probable cause to search the trunk of the car.
D) The officer was only justified in searching an area within the wingspan of the dealer once he was out of the car.

A

C) The officer lacked probable cause to search the trunk of the car.

The Fourth Amendment does not require police to obtain a warrant to search a vehicle if they have probable cause to believe that it contains contraband or evidence of criminal activity. The police may search anywhere in a car that they have probable cause to believe contains contraband, including the trunk and locked containers.

Here, the officer smelled marijuana smoke, which gave him probable cause to search the passenger compartment of the car. However, the court could have logically concluded that the officer lacked probable cause to believe that the smoke was emanating from the trunk of the car, since the dealer, while driving, would not have had access to the trunk of the car, and there was no one else in the car

75
Q

Two police officers stopped a car for a minor traffic violation. While one of the officers dealt with the driver and the traffic violation, the other officer talked with the passenger. Seeking to question the passenger about gang involvement, but lacking reasonable suspicions of criminal activity, the officer ordered the passenger out of the car. As the passenger exited the car, the officer saw a bulge in the passenger’s coat which the officer suspected might be a gun. Upon patting down the passenger, the officer felt the handle of a revolver and removed the gun. The passenger, who was a convicted felon, was charged with the possession of a gun by a prohibited possessor. Prior to trial, the defendant sought to suppress the gun as evidence, contending that its seizure was unconstitutional.

Should the court suppress the gun as evidence?

A) Yes, because, since the car had been stopped for a traffic violation, the officer could not order the passenger, who had not committed the violation, to exit the car.
B) Yes, because, since the officer lacked reasonable suspicion that the passenger was engaged in criminal activity, the officer could not pat down the passenger.
C) No, because a valid traffic stop gives an officer the right to pat down a passenger.
D No, because the gun was discovered as a consequence of a valid Terry stop and frisk.

A

D No, because the gun was discovered as a consequence of a valid Terry stop and frisk.

The traffic violation gave the police officers a valid reason for stopping the car. As part of the stop, the police officer could order the passenger to exit the vehicle. As the passenger did so, the police officer gained a reasonable suspicion that the passenger was armed and dangerous. Consequently, the officer could pat down the passenger for weapons.

76
Q

At a defendant’s trial for bank robbery, the prosecutor called a witness who claimed to be in the bank at the time of the robbery and identified the defendant as one of the bank robbers. The witness had previously identified the defendant from a lineup of six photographs shortly after the robbery, all of which were properly entered into evidence. The defendant’s attorney had not been present at the lineup. Seeking to impeach the witness with a potential bias, the defense attorney asked the witness on cross-examination whether he was affiliated with a gang that rivaled the defendant’s own. The witness refused to answer by invoking his privilege against self-incrimination. The judge allowed the witness to claim this privilege. The defense attorney then moved to strike the witness’s testimony, including his identification of the defendant, from the record.

Of the following, which argument best supports the request to strike the witness’s testimony?

A) The photo lineup violated the defendant’s right to have counsel present at all critical stages of his prosecution.
B) The prosecution violated the defendant’s privilege against self-incrimination by using his photograph in the lineup.
C) The witness’s interest in the privilege is outweighed by a criminal defendant’s right to defend himself.
D) The witness’s invocation of privilege to avoid this impeachment violates the defendant’s right to confrontation.

A

D) The witness’s invocation of privilege to avoid this impeachment violates the defendant’s right to confrontation.

A witness may invoke the right against self-incrimination in response to specific questions. However, such an invocation after testimony has already been made may violate a defendant’s right to confrontation, guaranteed by the Sixth and Fourteenth Amendments. A denial of the opportunity to cross-examine a prosecution witness with regard to bias violates the Confrontation Clause.

Note: The witness’s privilege against self-incrimination is not outweighed by the defendant’s own constitutional rights.

77
Q

A defendant was in jail for an aggravated assault charge after his attorney was unable to post bail for that offense. The police believed that the defendant had also been involved with other criminals in executing an unrelated series of bank robberies, and they placed an undercover officer in the jail to pose as the defendant’s cellmate. They hoped to acquire information that might prevent further robberies from occurring. One afternoon, the undercover officer mentioned that he knew the victim of the assault with which the defendant had been charged. The defendant said that the victim had a beating coming to him, and that he was glad he had been the one to complete that task. The next day, the undercover officer mentioned the bank robberies. The defendant began bragging that he had a substantial amount of cash waiting for him once he got out of jail, noting that “all those bankers would hardly miss it.”

Which of the defendant’s statements to the undercover officer are admissible against him at trial?

A) The statements regarding the assault and the bank robbery are both admissible.
B) The statement regarding the assault is admissible, but the statement regarding the robbery is not admissible.
C) The statement regarding the robbery is admissible, but the statement regarding the assault is not admissible.
D) Neither the statement regarding the assault nor the statement regarding the bank robbery is admissible.

A

C) The statement regarding the robbery is admissible, but the statement regarding the assault is not admissible.

Once the Sixth Amendment right to counsel attaches, statements that a defendant makes to a police informant are inadmissible when the police intentionally create a situation likely to induce the defendant into making incriminating statements without the assistance of counsel. In this case, the police created a situation likely to induce the defendant into making a statement about the assault without the presence of counsel; that statement would thus be inadmissible

That protection would not apply to the statement about the robbery, however. Under the Sixth Amendment offense-specific standard, the requirement for counsel to be present applies only to interrogations about the offense charged. In this case, the defendant’s Sixth Amendment right to counsel did not apply to questioning about the bank robberies.

78
Q

One evening, a man was arrested for robbery. From jail, he called his sister and told her that he had been arrested, but made no mention of hiring a lawyer. The sister nevertheless employed a lawyer to represent the man. The lawyer contacted the police, told them that she represented the man, and requested to be present when the man was questioned. The police lied to the lawyer, telling her that the man had fallen asleep and would not be questioned that night. Instead, after giving the man his Miranda warnings, the police questioned the man for two hours before he voluntarily confessed to the crime. Although the man did not request a lawyer during his interrogation, the police did not tell him that his sister had secured one for him or that his lawyer had asked to be present during any interrogation. The next morning at his arraignment, the man was formally charged with robbery.

Should the man’s confession be excluded as a violation of his Sixth Amendment right to counsel?

A) No, because he had not yet been arraigned when he confessed.
B) No, because he waived this right by voluntarily confessing after receiving Miranda warnings.
C) Yes, because a lawyer had been employed to represent the man before he confessed.
D) Yes, because the police lied to the man’s lawyer and failed to inform the man that his lawyer had requested to be present during his interrogation.

A

A) No, because he had not yet been arraigned when he confessed.

After receiving Miranda warnings, a defendant may waive his Sixth Amendment right to counsel, as well as his Fifth Amendment right to counsel. In this case, however, the man’s Sixth Amendment right to counsel had not attached.

79
Q

After a defendant was indicted and arraigned on a robbery charge, the defendant was required to participate in a lineup at a police station. The defendant’s lawyer was not informed of the lineup and the defendant did not waive his right to have his lawyer present. The victim of the robbery identified the defendant as the robber. At trial, the victim again identified the defendant as the robber. When asked about her identification, she stated that she was sure that the defendant was the robber because she remembered the mole on his right cheek. The defendant’s lawyer moves to exclude this testimony as obtained in violation of the defendant’s Sixth Amendment right to counsel.

Is the court likely to grant this motion?

A) No, because the witness’s identification was not based on the defendant’s participation in the lineup.
B) No, because the witness is testifying at the defendant’s trial and is available for cross-examination.
C) Yes, because a defendant’s Sixth Amendment right to counsel applies to a post-indictment lineup.
D) Yes, because the defendant’s lawyer was not present at the lineup and the defendant did not waive his right to have his lawyer present.

A

A) No, because the witness’s identification was not based on the defendant’s participation in the lineup.

If a right to counsel at a trial proceeding under the Sixth Amendment is denied, the defendant’s conviction should be automatically reversed, even without a specific showing of unfairness.

However, a denial of counsel at a nontrial proceeding, such as a lineup, is subject to a harmless-error analysis. Here, the witness’s identification of the defendant at trial is based on her memory of the actual event rather than the lineup.

80
Q

A state prosecutor filed an information charging a man with first-degree arson and first-degree murder. In the applicable jurisdiction, first-degree arson is defined as “the willful and malicious setting fire to and burning of a dwelling.” After the presentation of the prosecution’s case, the man’s counsel moved for a dismissal of the arson charge because the prosecution had failed to prove that the structure that was burned was a dwelling. The court granted this motion. However, because the evidence presented by the prosecution established second-degree arson, the court permitted the prosecution to amend the information to charge the man with second-degree arson. The man then took the stand and, while testifying, admitted that the structure was a dwelling. The court permitted the prosecution to again amend the information to charge the man with first-degree arson. The jury subsequently convicted the man of first-degree arson.

Can the man successfully challenge his conviction on constitutional grounds?

A) No, because the man’s testimony caused the reinstatement of the first-degree arson charge.
B) No, because the trial was not terminated by the dismissal of the first-degree arson charge.
C) Yes, because the court dismissed the first-degree arson charge against the man.
D) Yes, because the man was not charged with arson through an indictment.

A

C) Yes, because the court dismissed the first-degree arson charge against the man.

Under the Double Jeopardy Clause of the Fifth Amendment, which is applicable to the states through the Due Process Clause of the Fourteenth Amendment, a defendant who has been acquitted of a crime generally may not be retried for the same crime. A grant of a demurrer or motion to dismiss in favor of the accused for the prosecution’s failure to prove the elements of a crime at the close of the state’s case is the equivalent to an acquittal. In this case, the man was effectively acquitted of first-degree arson when the charge was dismissed after the presentation of the prosecution’s case.

81
Q

A man was set up on a blind date with a woman through a mutual friend. After the date, the man raped the woman; he was subsequently convicted of rape by a jury. After the trial, the woman’s injuries from the rape developed into an undetected hemorrhage that caused her death. A grand jury subsequently indicted the man for felony murder of the woman. The man filed a motion to dismiss the indictment because it violated his rights under the Double Jeopardy Clause.

Should the motion to dismiss be granted?

A) Yes, because the second prosecution may result in multiple punishments for the same offense.
B) Yes, because the man has already been convicted of rape for his conduct.
C) No, because rape and felony murder do not require proof of the same elements.
D) No, because the man is being indicted for a more serious crime, an element of which occurred after his rape conviction.

A

D) No, because the man is being indicted for a more serious crime, an element of which occurred after his rape conviction.

When jeopardy has attached with respect to a lesser included offense prior to the occurrence of an event necessary to establish the greater offense, the defendant may be subsequently tried for the greater offense

82
Q

A divorced man was considering entering his ex-wife’s house and taking some rare books that the ex-wife had bought after the divorce. The divorced man thought that his portion of the couple’s assets after the divorce had been inadequate, and that taking the books would make up the difference. Unsure about whether doing so was legal, he decided to consult an attorney. The attorney told the divorced man that, given the unfairness of the divorce settlement, it was fine to take the books. Late one night while she was out of town, the divorced man broke a window and entered his ex-wife’s home. He took the rare books. A few days later, he was arrested for criminal trespassing.

Can the divorced man raise his attorney’s advice as a defense?

A) No, because of the attorney-client privilege.
B) No, because incorrect advice from an attorney is not itself a defense.
C) Yes, because incorrect advice from an attorney is a defense.
D) Yes, because his mistake of law was reasonable.

A

B) No, because incorrect advice from an attorney is not itself a defense.

Note: Even if the divorced man’s mistake of law was reasonable, reliance on incorrect or bad legal advice from an attorney is not itself a valid mistake-of-law defense.

83
Q

A mother got a surprise visit from her son, a criminal on the run. Her son asked her to give him her gun so that he could kill a drug dealer who was pursuing him. He told her that the drug dealer would kill him if he did not kill the drug dealer first. The mother, who loved her son more than anything, gave him her gun and a package of bullets. The next day, the son went to another city, found the drug dealer, and shot and killed him. A few days later, the police found the son hiding in a shed in a friend’s backyard and arrested him for the drug dealer’s murder. They then traced the gun’s ownership to the mother and charged her with murder as well. The son was later convicted of the drug dealer’s murder.

Can the mother be convicted of murdering the drug dealer?

A) No, because the son, not the mother, shot the drug dealer.
B) No, because mere knowledge that a crime may occur cannot support a murder conviction.
C) Yes, because she gave her son the murder weapon.
D) Yes, because the son was convicted of murder.

A

C) Yes, because she gave her son the murder weapon.

Under the majority and MPC rule, an accomplice is a person who, with the purpose of promoting or facilitating the commission of the offense, aids or abets a principal prior to or during the commission of the crime. An accomplice to the crime can be convicted of the crime, even if he was not involved in the principal’s criminal actions. An accomplice is responsible for the crime to the same extent as the principal

Note: The mere fact that the son was convicted of murder is not enough to convict the mother. Her liability as an accomplice must be established.

84
Q

An unmarried mother was her six-month-old child’s sole caretaker. The mother left the child unattended for five days, during which time the mother was drunk. Her intoxication prevented her from appreciating the high risk of death that her inattention posed for her child. The child died from dehydration.

What is the most serious homicide offense of which the mother could be properly convicted?

A) Murder
B) Voluntary manslaughter
C) Involuntary manslaughter
D) No form of criminal homicide

A

A) Murder

A defendant can be guilty of murder even when the defendant lacked the intent to kill or inflict serious bodily harm. An unintentional killing that results from reckless indifference to an unjustifiably high risk to human life is a depraved-heart murder. Here, the mother was recklessly indifferent to her child’s fate in leaving the six-month-old child unattended for five days.

Even though the mother did not appreciate the unjustifiably high risk of death that her inattention posed for her child, because her failure to appreciate the objective risk was due to her voluntary intoxication, she can be convicted of depraved-heart murder.

Note: Involuntary manslaughter requires more than ordinary negligence for tort liability and something less than the extremely negligent conduct required for depraved-heart murder.

85
Q

A baker who had a contract for blueberry muffins tried unsuccessfully to negotiate with a blueberry farmer to purchase blueberries. The following evening, the baker waited until the farmer left her roadside stand and then entered the farmer’s adjacent blueberry fields, and picked 25 quarts of blueberries. As the baker was loading the blueberries into his van, the baker had second thoughts about what he had done. Realizing the farmer would have no problem selling the blueberries at his stand, the baker carried the buckets filled with blueberries to the front of the farmer’s stand. A few moments later, he was apprehended and charged with larceny.

Of the following, which would provide baker with the best defense?

A) The baker had begun to voluntarily return the blueberries before he was apprehended.
B) The baker had not permanently deprived the farmer of her blueberries.
C) The farmer would not have suffered a loss because she could have sold the blueberries at her stand.
D) The baker had picked the blueberries himself.

A

D) The baker had picked the blueberries himself.

Larceny elements:
1. Trespassory
2. Taking
3. of another’s personal property
4. With the intent to deprive that person of the property

The taking of real-property items (e.g., unharvested crops) is not larceny when the defendant’s act of severance occurs immediately before the carrying away of the real-property items.

86
Q

A mother was in the hospital with a terminal illness. One of her daughters went to visit her, and they became engaged in a conversation about the mother’s engagement ring, which the daughter and her sisters had long admired. The mother told her daughter that she planned to leave the ring to the daughter’s sister. Angered by her mother’s decision, the daughter took the ring from her mother’s bedside table that night after her mother fell asleep. As soon as she got to her car, however, the daughter had a change of heart and returned the ring to her mother’s table. Later that day, the mother died. Contrary to her statement, she left the ring to the daughter in her will.

Could the daughter be convicted of any of the following crimes?:

A) Attempted larceny only.
B) Larceny only.
C) Larceny and attempted larceny.
D) The daughter may not be convicted of any theft crime.

A

B) Larceny only.

Larceny:
1) trespassory
2) taking and carrying away
3) personal property of another
4) with the intent to permanently deprive that person of the property.

The elements of larceny are all met in this case.

Although the woman became the rightful owner of the ring shortly after she took it, the ring still belonged to her mother* at the time it was taken.

87
Q

A man and a woman agreed to engage in consensual sexual intercourse at a particular hotel the following night. The man, who was the son of the woman’s brother-in-law, rented the hotel room for their tryst, but the woman did not show up.

In the applicable jurisdiction, a statute provides that a person commits the offense of incest when such person engages in sexual intercourse with a person whom he or she knows he or she is related to either by blood or by marriage as aunt or nephew.

Can the woman and man properly be charged with conspiracy to commit incest?

A) Yes, because each consented to engage in sexual intercourse.
B) Yes, because the man rented a hotel room.
C) No, because of the Wharton Rule.
D) No, because the woman did not show up to the hotel.

A

C) No, because of the Wharton Rule.

Wharton Rule: if a crime requires two or more participants, there is no conspiracy unless more parties than are necessary to complete the crime agree to commit the crime.

Here, only the woman and man agreed to commit incest and both are required in order to commit this crime, neither can properly be charged with the crime of conspiracy to commit incest

88
Q

Several days after learning his business partner had swindled him out of a large sum of money, a man bound and gagged the business partner and forced him into the man’s car at gunpoint. The man then drove to the home of the business partner’s accomplice. The man got out of the car and yelled for the accomplice to come outside. The man stood waiting in front of the front passenger door of the car, blocking the business partner from the view of anyone in the residence. The accomplice, looking out a window and seeing only the armed man, grabbed a gun, and opened the front door. The man fired at the accomplice, but missed him. The accomplice shot at the man intending to kill him, but the man ducked and the bullet hit and killed the business partner. The accomplice was arrested and charged with the murder of the business partner.

Is the accomplice likely to be convicted of this crime?

A) No, because the accomplice acted in self-defense.
B) No, because the accomplice lacked the intent to kill the business partner.
C) Yes, under the doctrine of transferred intent.
D) Yes, because the business partner would not have died but for the accomplice’s act.

A

A) No, because the accomplice acted in self-defense.

One who is not the aggressor is justified in using reasonable force against another person to prevent immediate unlawful harm to himself. Deadly force may be justified in self-defense only when it is reasonably necessary to prevent death or serious injury or to prevent the commission of a serious felony involving a risk to human life. Here, the accomplice fired at the man in self-defense as the man had fired at the accomplice

89
Q

A defendant was lawfully arrested for bank robbery, without an arrest warrant. He was not given Miranda warnings but was immediately taken to a police station, where he and five other men were placed in a lineup to be viewed by the bank teller who had been on duty at the time of the robbery. Each man was required to say the words spoken by the bank robber: “Give me all your money. I’ve got a gun.” After all the men in the lineup spoke those words, the teller identified the defendant as the robber.
The defendant subsequently moved to suppress the teller’s testimony, claiming that the lineup violated his privilege against self-incrimination. At the suppression hearing, the teller testified that she had not gotten a good look at the robber’s face, because the robber had been wearing a hat pulled down over most of his face, but that she was certain the defendant was the robber because she had recognized his voice at the lineup.

Should the defendant’s motion be granted?

(A) No, because being required to speak at the lineup, while compelled, was not testimonial or communicative.
(B) No, because testimony of a witness based on firsthand observation is not subject to exclusion as the fruit of the poisonous tree.
(C) Yes, because the defendant was compelled to speak at the lineup, and this compelled speech led to the witness’s identification testimony.
(D) Yes, because the defendant was never informed that he could refuse to make a statement and that any statement could be used as evidence against him.

A

(A) No, because being required to speak at the lineup, while compelled, was not testimonial or communicative.

The defendant properly could be required to utter the words spoken by the bank robber. The privilege against self-incrimination extends only to compelled “testimonial” communications; “[t]hus, even though the act may provide incriminating evidence, a criminal suspect may be compelled . . . to make a recording of his voice.

Note: Requiring the defendant to utter the words spoken by the bank robber was not a compelled “testimonial” communication. Accordingly, it was not necessary to provide Miranda warnings, which are designed to protect the privilege against self-incrimination.

90
Q

An ordinance in a small town required all restaurants to designate smoking and nonsmoking sections for their customers. A cigarette smoker and a nonsmoker were seated at adjoining tables in a small restaurant. The smoker’s table was in the smoking section, and the nonsmoker’s table was in the nonsmoking section. When the smoker lit a cigarette, the nonsmoker politely requested that he not smoke, explaining that she had a severe allergy to cigarette smoke. The smoker ignored the nonsmoker’s request and continued to smoke. As a result, the nonsmoker was hospitalized with a severe allergic reaction to the smoke.
The nonsmoker brought a battery action against the smoker.
Which of the following questions will NOT be an issue in the battery action?

(A) Did the smoker intend to cause the nonsmoker’s contact with the cigarette smoke?
(B) Does smoke have the physical properties necessary for making the kind of contact required for battery?
(C) Is contact with cigarette smoke from a lawful smoking section in a restaurant the kind of contact one must endure as a voluntary restaurant patron?
(D) Was the smoker’s conduct unreasonable under the circumstances?

A

(D) Was the smoker’s conduct unreasonable under the circumstances?

Whether a battery defendant’s conduct was reasonable under the circumstances is irrelevant if in fact the defendant intended to make a harmful or offensive contact with the plaintiff. It would be relevant in a negligence action, but not in a battery action.

Note: The plaintiff in a battery action must establish that some sort of bodily contact occurred. It is debatable whether smoke is sufficiently physical and forceful to create the necessary touching.

91
Q

United States customs officials received an anonymous tip that heroin would be found inside a distinctively marked red package mailed from a foreign country to a particular address in the United States. Pursuant to this tip, United States customs officers intercepted and opened the red package and found heroin inside. They then resealed the package and left the heroin inside it. The FBI was notified and, as agents watched, the package was delivered to the address.
The FBI then secured a warrant to search the house for the package. About two hours after the package was delivered, agents executed the warrant at the house. The man who opened the door was arrested, and the agents found the package, unopened, in an upstairs bedroom closet. After seizing the package, the agents looked through the rest of the house. They found a machine gun in a footlocker in the basement.
The man was charged with unlawful possession of the machine gun, among other crimes. He moved to suppress the use of the gun as evidence.

Should the court grant the motion to suppress the machine gun?

A) Yes, because the search exceeded the authority granted by the warrant.
(B) Yes, because the initial search by the customs officers was without probable cause.
(C) No, because, having found the package, the agents had probable cause to believe that more narcotics could be located in the house and the gun was found in a proper search for narcotics.
(D) No, because narcotics dealers are often armed and the search was justified to protect the agents.

A

A) Yes, because the search exceeded the authority granted by the warrant.

The warrant was valid, but its validity was triggered by and limited to the delivered package. Accordingly, once the only object of that search was discovered, the warrant did not authorize a further exploratory search of the house.

Note: The search warrant was valid, but its validity was triggered by and limited to the delivered package. Accordingly, once the only object of that search—the package—was discovered, the warrant did not authorize a further exploratory search of the house.

92
Q

In a city, a number of armed bank robberies were committed near closing time by a masked man wearing a white hooded sweatshirt and blue sweatpants. Police saw a man wearing a white hooded sweatshirt and blue sweatpants pacing nervously outside one of the city’s banks just before it closed. The police stopped the man and frisked the outer layers of his clothing for weapons, but found none. They asked the man what he was doing outside the bank and pointed out that he was wearing clothing similar to clothing worn by the perpetrator of recent robberies. After pausing for several moments, the man confessed. The police had not provided him with any Miranda warnings.
After being charged with the bank robberies, the man moved to suppress his confession. The parties agreed, and the court properly found, that the police had reasonable suspicion, but not probable cause, at all times before the man confessed.
Should the man’s motion to suppress be granted?

(A) Yes, because the confession was the fruit of a Fourth Amendment violation, even though there was no Miranda violation.
B) Yes, because the confession was the fruit of a Miranda violation, even though there was no Fourth Amendment violation.
(C) Yes, because the confession was the fruit of both a Fourth Amendment violation and a Miranda violation.
(D) No, because there was neither a Fourth Amendment violation nor a Miranda violation.

A

(D) No, because there was neither a Fourth Amendment violation nor a Miranda violation.

There was neither a Fourth Amendment violation nor a Miranda violation. There was no Fourth Amendment violation because the stop, frisk, and questioning were permissible, under Terry v. Ohio, 392 U.S. 1 (1968), based on reasonable suspicion. There was no Miranda violation because warnings are not required for Terry stops

93
Q

A drug dealer agreed with another individual to purchase heroin from the individual in order to sell it on a city street corner. Unbeknownst to the drug dealer, the other individual was an undercover police officer whose only purpose was to arrest distributors of drugs. The drug dealer made a down payment for the heroin and agreed to pay the remainder after he sold it on the street. As soon as the undercover officer handed over the heroin, other officers moved in and arrested the dealer.
The jurisdiction follows the common law approach to conspiracy.

Could the dealer properly be convicted of conspiring to distribute drugs?

(A) No, because there was no overt act.
(B) No, because there was no plurality of agreement.
(C) Yes, because neither an overt act nor plurality of agreement is required at common law.
(D) Yes, because the dealer believed that all the elements of conspiracy were present and cannot take advantage of a mistake of fact or law.

A

(B) No, because there was no plurality of agreement.

The common law requires plurality of agreement and does not criminalize “unilateral” conspiracy where only one person actually agreed to commit the crime and the other only feigned agreement.

Note: While the common law does not require an overt act, the dealer could not properly be convicted for conspiring with an undercover officer.

94
Q

A foreign diplomat discovered that a small person could enter a particular jewelry store by crawling through an air vent. The diplomat befriended a woman he met in a bar who he believed was small enough to crawl through the air vent. Without telling her that he was a diplomat, he explained how she could get into the jewelry store. She agreed to help him burglarize the store. Someone overheard their conversation and reported it to the police. Shortly thereafter, the police arrested the diplomat and the woman. Both were charged with conspiracy to commit burglary.
Before trial, the diplomat moved to dismiss the charge against him on the ground that he was entitled to diplomatic immunity. The court granted his motion. The woman then moved to dismiss the conspiracy charge against her.

The jurisdiction has adopted the Model Penal Code version of conspiracy.

Should the court grant the woman’s motion to dismiss the conspiracy charge against her?

(A) No, because the diplomat’s defense does not negate any element of the crime.
(B) No, because the woman was not aware of the diplomat’s status.
(C) Yes, because a conspiracy requires two guilty participants.
(D) Yes, because but for the diplomat’s conduct, no conspiracy would have occurred.

A

(A) No, because the diplomat’s defense does not negate any element of the crime

MPC = conspiracy requires agreement by the defendant but not by two or more persons. In addition, many jurisdictions that require a bilateral conspiracy still allow conviction if one of the co-conspirators agreed to the crime but cannot be convicted based on lack of capacity or some other defense personal to the co-conspirator.

95
Q

A man decided to steal a car he saw parked on a hill. After he got in and started the engine, the car began rolling down the hill. The man quickly discovered that the car’s brakes did not work. He crashed through the window of a store located at the bottom of the hill.
The man was charged with larceny of the car and with the crime of malicious damage to property. At trial, the judge instructed the jury that if the jury found both that the man was guilty of larceny of the car and that the damage to the store was the result of that larceny, then it should also find him guilty of malicious damage to property.
The man was convicted on both counts. On appeal, he argued that the conviction for malicious damage to property should be reversed because the instruction was not a correct statement of the law.

Should the man’s conviction be affirmed?

(A) Yes, because his intent to steal the car provides the necessary mental element.
(B) Yes, because he was committing a felony.
(C) No, because the instruction wrongly described the necessary mental state.
(D) No, because it would violate double jeopardy to convict the man of two crimes for a single act.

A

C) No, because the instruction wrongly described the necessary mental state.

The instruction was wrong, because to have acted knowingly with respect to the malicious damage charge, the man must have been practically certain that his conduct would damage the store.

Note: Intent to steal the car would not prove that the man knowingly damaged the store. To have acted knowingly with respect to the malicious damage charge, the man must have been practically certain that his conduct would cause damage.

96
Q

A store clerk had been working at a convenience store for four years when a new owner took over. On the second day that the new owner was at the store, she fired the clerk. The clerk was so angry that he decided to kill the new owner. However, he knew that the store was frequented by police officers, and he was afraid of being seen, caught, arrested, or even shot. When he arrived home, the clerk told his girlfriend, whom he knew to be vengeful and easily provoked to violence, that the new owner had made sexual advances towards him. The girlfriend became extremely angry and said, “I want to kill her.” The clerk said, “I would, too, if I were you. I’d kill her so fast she wouldn’t know what hit her.” The girlfriend grabbed a large machete from the garage and headed to the store. As she was charging into the store with the machete raised toward the store owner, a police officer outside the store saw her and, believing she was going to kill the owner, shot her. The girlfriend died from the gunshot.

If the state charges the clerk with the murder of the girlfriend, is he likely to be convicted?

A) No, because he never intended his girlfriend to be shot.
B) No, because the Redline doctrine prevents his conviction for the girlfriend’s death.
C) Yes, because he acted with a depraved heart.
D) Yes, because his intent to kill the store owner transferred to the girlfriend.

A

A) No, because he never intended his girlfriend to be shot.

Murder: unlawful killing of another committed with malice aforethought.

Here, the store clerk did not intend for his girlfriend to be shot. BUT he did know that the convenience store was frequented by police officers and that the girlfriend was vengeful and easily provoked to violence.

Therefore, the store clerk acted recklessly by lying to the girlfriend about the store owner making sexual advances toward him and encouraging her desire to kill the store owner

97
Q

A collector of historical documents discovered that a letter she had purchased many years earlier that she thought was in the handwriting of a famous historical figure was a forgery. Because the person who sold her the letter was now dead, the collector advertised the letter for sale as genuine and priced it accordingly. The benefactor of a museum expressed interest in purchasing the letter. But knowing that forgeries of the historical figure’s letters were common and not trusting the collector’s word, the benefactor had a curator at the museum examine the letter. The curator mistakenly informed the benefactor that the letter was genuine. As a consequence, the benefactor purchased the letter, paying the collector in cash.

Can the collector be properly charged with false pretenses?

A) No, because the benefactor did not rely on the collector’s representation of the letter as genuine.
B) No, because the collector did not forge the letter.
C) Yes, because the benefactor paid the collector for the letter.
D) Yes, because the collector advertised and priced the letter as genuine.

A

A) No, because the benefactor did not rely on the collector’s representation of the letter as genuine.

False Pretenses: the prosecution must prove that the defendant’s misrepresentation
1) was a significant factor in,
2) or the cause of, the victim’s decision to pass title to the property to the defendant.

98
Q

In response to a request by the president of a local sorority, the owner of a liquor store agreed to supply beer to the sorority for a party at a 10 percent discount off the regular price. The owner was aware that a number of sorority members were under the age of 21, but he did not intend to supply alcoholic beverages to individuals under the age of 21. The owner delivered the beer to the sorority, and the president of the sorority, who was 21 years old, paid for the beer. The sorority permitted anyone who attended the party, including those under 21 years of age, to drink the beer supplied by the owner.

By statute, it is a criminal offense if a person supplies an alcoholic beverage to an individual under 21 years of age. This statute has been interpreted as a strict liability offense.

Can the owner properly be charged with conspiracy to supply alcoholic beverages to individuals under the age of 21?

A) No, because a merchant of goods cannot be criminally liable for legally selling such goods.
B) No, because the owner did not have the specific intent to supply alcoholic beverages to individuals under the age of 21.
C) Yes, because the offense of supplying beer to individuals under age 21 has been interpreted as a strict liability offense.
D) Yes, because the owner should have been aware that there was a substantial and unjustifiable risk that the beer would end up in the hands of individuals who were under the age of 21.

A

B) No, because the owner did not have the specific intent to supply alcoholic beverages to individuals under the age of 21.

A conspiracy conviction requires proof that the defendant had the specific intent to accomplish an unlawful purpose—even if the intended crime does not require specific intent (e.g., when the intended crime is a strict liability offense).

99
Q

A loan shark instructed a henchman to either collect a debt from a borrower or shatter his kneecaps as a message to other borrowers on the importance of timely repayment of their loans. The henchman, mistaking the borrower’s twin brother for the borrower, shattered the twin’s kneecap when the twin refused to pay back a debt he had never incurred. The twin died from the shattered kneecap when a fragment of bone severed an artery and he bled to death.

Is the loan shark likely to be found guilty of attempted murder of the borrower?

A) No, because the borrower was not ultimately harmed.
B) No, because the loan shark did not intend for the shattering of the borrower’s kneecaps to result in his death.
C) Yes, because the loan shark is vicariously liable for the acts of his agent, the henchman.
D) Yes, under the doctrine of transferred intent.

A

B) No, because the loan shark did not intend for the shattering of the borrower’s kneecaps to result in his death.

Specific intent is required for an attempt conviction even if liability for the completed crime can be imposed without that intent.

100
Q

A police officer submitted a warrant application to search a particular residence for methamphetamines (meth). The application included the officer’s affidavit that a known, but unidentified, reliable informant had told the officer that on the previous evening the informant had purchased meth from the occupant of the residence and had observed paraphernalia for the manufacture of meth. The magistrate issued a warrant for a search of the residence and for seizure of meth and related manufacturing paraphernalia, and police searched the residence. The search did not reveal any meth or related manufacturing paraphernalia, but it did uncover cocaine. The occupant of the residence was arrested and charged, based on the quantity of cocaine found at the residence, with distribution of cocaine.

The occupant challenged the admissibility of the cocaine on the ground that the informant had not been at the residence on the evening before the warrant was issued. At the suppression hearing, the judge concluded that the informant had lied to the police officer but that the officer had not known of the falsity of the informant’s statements at the time that the warrant was issued.

On these facts and findings, is the judge likely to suppress the cocaine?

A) No, because a defendant may not challenge a facially valid warrant on the basis of a lack of probable cause.
B) No, because the police officer did not know the informant’s statements were false at the time that the warrant was issued.
C) Yes, because the informant lied to the police officer and his false statements served as the basis for the issuance of the warrant.
D) Yes, because the warrant did not authorize the police to seize cocaine.

A

B) No, because the police officer did not know the informant’s statements were false at the time that the warrant was issued.

A search warrant is invalid if
(1) the application contained a false statement necessary to the finding of probable cause and
(2) the officer knew that the statement was false or recklessly disregarded its falsity.

101
Q

After receiving a grant of use and derivative use immunity, a mayor testified before a grand jury regarding allegations of corruption. Subsequently, the mayor was indicted for the crime of extortion by a public official based on evidence that the prosecutor obtained without the use of the mayor’s grand jury testimony. At trial, the mayor testified on his own behalf.

Is it constitutionally proper for the prosecutor to impeach the mayor’s testimony with a statement that the mayor made before the grand jury?

A) No, because the mayor did not waive his Fifth Amendment privilege against self-incrimination by taking the stand.
B) No, because the mayor’s grand jury testimony was coerced by the grant of immunity.
C) Yes, because statements otherwise protected by the Fifth Amendment may be used for impeachment purposes.
D) Yes, because the prosecutor indicted the mayor without the use of the mayor’s grand jury testimony.

A

B) No, because the mayor’s grand jury testimony was coerced by the grant of immunity.

Testimony given under a grant of immunity is coerced and therefore involuntary. Any criminal-trial use of such testimony outside the context of a perjury prosecution (e.g., for impeachment) is a denial of due process.

102
Q

A state official was charged with two federal offenses—bribery in connection with a program administered by the state official that receives federal funds, and interstate travel in furtherance of bribery. The bribe allegedly consisted of accepting payment of the expenses of an out-of-state trip taken by the state official in exchange for awarding a grant under the federal program to the payor of the expenses. The jury found the state official not guilty of bribery but guilty of the offense of interstate travel in furtherance of bribery. The state official has challenged his conviction for interstate travel in furtherance of bribery as unconstitutional.

How should the court rule on this challenge?

A) Against the state official, because a jury verdict is not subject to challenge on appeal.
B) Against the state official, because acquittal by a jury of a predicate offense does not mandate acquittal of a related offense.
C) For the state official, because his conviction for interstate travel in furtherance of bribery is inconsistent with acquittal of bribery.
D) For the state official, because the bribery offense was a lesser included offense of the travel offense.

A

B) Against the state official, because acquittal by a jury of a predicate offense does not mandate acquittal of a related offense.

A guilty verdict cannot be challenged on the ground that it is inconsistent with another verdict rendered by the same jury.

Here, the jury found the state official not guilty of bribery but guilty of interstate travel in furtherance of bribery. Those verdicts are inconsistent because interstate travel in furtherance of bribery is predicated upon the actual commission of bribery.

BUT since acquittal of a predicate offense does not mandate acquittal of a related offense, the state official cannot properly challenge his conviction on the basis of inconsistency.

103
Q

In state court, a woman was found guilty of misdemeanor possession of a controlled substance, and the court imposed a sentence of six months’ imprisonment. The execution of the sentence was suspended, and she was placed on probation for six months. The woman later violated a condition of her probation by failing to attend a treatment program. The woman admitted her violation at a preliminary probation-revocation hearing, but at her final probation-revocation hearing she made the necessary proffer of indigence and a timely request for the appointment of counsel, which the court denied. At this final hearing, the court, as permitted by state law, revoked her probation and executed the sentence of six months for violation of the terms of her probation.

Did the woman have a constitutional right to the appointment of counsel at the final probation hearing?

A) No, because execution of the woman’s sentence was triggered by revocation of her probation.
B) No, because the court’s sentence did not exceed six months’ imprisonment.
C) Yes, because an indigent defendant has the right to the appointment of counsel.
D) Yes, because the court’s action resulted in the woman’s incarceration.

A

A) No, because execution of the woman’s sentence was triggered by revocation of her probation.

Due process requires counsel at a probation-revocation proceeding if the person
(1) denies committing the alleged violation or
(2) asserts that complex reasons justified or mitigated it.

The Sixth Amendment guarantees this right if a sentence for the underlying offense (1) has not been imposed and (2) will be imposed if the probation is revoked.

Here, a sentence of six months was imposed on the woman after she was found guilty of possessing a controlled substance. It was merely the execution of that sentence that had been suspended.

104
Q

An 18-year-old defendant was arrested for armed robbery. After being Mirandized, the defendant asked for a lawyer. The police did nothing about this request, but instead contacted the defendant’s parents who came to the police station where the defendant was being held. The police placed the defendant and his two parents in a bugged room in the hopes that the defendant would incriminate himself. The defendant confessed to his parents that he committed the armed robbery. Learning that the prosecution now plans to use the confession at the defendant’s trial, the defendant’s attorney has filed a motion to suppress this confession as a violation of the defendant’s Fifth Amendment right to counsel.

Of the following, which represents the best factual support for the defense’s motion?

A) The defendant asked for an attorney after being Mirandized.
B) The defendant did not know that the room was bugged.
C) The police placed the defendant and his parents in a room in the hopes that the defendant would incriminate himself.
D) The police took unfair advantage of the defendant’s voluntary statements to his parents

A

C) The police placed the defendant and his parents in a room in the hopes that the defendant would incriminate himself.

Once the right to counsel is invoked, the defendant may not be subjected to further interrogation without counsel present. Interrogation refers not only to express questioning, but also to any words or actions that the police know or should know are likely to elicit an incriminating response.

Here, the police appear to have arranged for the defendant to meet with his parents in an effort to obtain an incriminating statement. Therefore, this would be the defendant’s best argument for excluding the statement

Note: the police’s failure to comply with the student’s request for an attorney, by itself, will not make the statement inadmissible.

1) Was D in custody? YES
2) Was there an interrogation? YES (see above)
3) Was D given Miranda warnings? YES
4) Did D invoke his rights? YES – yet the police continued with the interrogation

105
Q

A college student was inducted into a secret society. After an oath ceremony, the active members told the inductee that they were planning to raid an animal lab and set all of the animals free. The inductee agreed to help and told the active members how noble their mission was. He encouraged them to bring as many people as possible to the animal lab, all the better to set more animals free. The active members and the inductee agreed to meet at the animal lab at midnight. At 11:00 p.m., the inductee decided not to join the secret society. He did not go to the animal lab, and he did not communicate his decision to the active members. He decided that they would know he had changed his mind when he did not show up. As planned, the active members broke into the animal lab at midnight and set the animals free. The next day, the police arrested the inductee for burglary. They told him that the active members had turned him in.

In a state that has expanded burglary by statute to include non-dwellings, will the inductee likely be convicted of burglary?

A) Yes, because he knew that the active members planned to break into the animal lab.
B) Yes, because he did nothing to stop the active members from carrying out the crime.
C) No, because he did not break into the animal lab to set all of the animals free.
D) No, because he decided not join the secret society.

A

B) Yes, because he did nothing to stop the active members from carrying out the crime.

To legally withdraw (and therefore avoid liability for the substantive crime), an accomplice must
(i) repudiate prior aid,
(ii) do all that is possible to countermand prior assistance, and
(iii) do so before the chain of events is in motion and unstoppable.

A mere change of heart, a flight from the crime scene, an arrest by law enforcement, or an uncommunicated decision to withdraw is ineffective

106
Q

A woman learned that a wine connoisseur had recently purchased a case of very expensive fine wine that he stored in a wine cellar in his basement. The woman received an invitation from the connoisseur to a party to be held at his house. On the night of the event, during the dinner, the woman excused herself to powder her nose, but instead sneaked down to the basement to grab a bottle of the wine for herself. She opened the door to the wine cellar and entered it. As she searched for a bottle of the fine wine, the connoisseur entered the wine cellar. The woman apologized for being there, and told the connoisseur that she couldn’t help but see his cellar. The woman left without taking any bottles of wine. The connoisseur later overheard the woman talking with a friend as they were leaving the party about the woman’s thwarted attempt to take a bottle of the fine wine.

Can the woman properly be charged with burglary?

A) Yes, because the woman intended to steal the wine from the wine cellar.
B) Yes, because the woman would have taken a bottle of the wine if the wine connoisseur had not found her in the cellar.
C) No, because the woman was a guest of the connoisseur’s.
D) No, because the woman did not take a bottle of wine from the connoisseur’s house.

A

A) Yes, because the woman intended to steal the wine from the wine cellar.

Here, the woman opened the door to the wine cellar and entered it with the intention of committing larceny by taking a bottle of the fine wine from the connoisseur’s wine cellar.

Note: even though a defendant gains entry into a dwelling with consent, a breaking can still occur if the defendant breaks into a part of the dwelling structure where the defendant lacks consent to be. Here, the woman, although in the connoisseur’s house at his invitation, exceeded the scope of his consent by entering the wine cellar

107
Q

Two men and a woman plotted to burn down a building. Before the men and woman took any overt act towards completion of the arson, the police discovered the plot and arrested the three plotters on charges of conspiracy to commit arson. For evidentiary reasons, the prosecutor decided that he could only charge the woman for the conspiracy and decided not to charge the two men.

The woman is convicted of conspiracy in a jurisdiction that follows the common law. On appeal, should the woman’s conviction be overturned?

A) Yes, because only one of the three alleged conspirators was actually charged with conspiracy.
B) Yes, because there was no overt act in furtherance of the conspiracy.
C) No, because she and the men plotted to burn down a building.
D) No, because of the “Pinkerton Rule.”

A

C) No, because she and the men plotted to burn down a building.

At common law, all that is required for a conspiracy to be established is an agreement between two or more persons to accomplish an unlawful purpose with the intent to accomplish that purpose. No overt act is required for the conspiracy to be complete at common law.

Note: the fact that all but one of the co-conspirators are not charged with conspiracy does not prevent a conviction of the conspirator who is charged, so long as the prosecution can show that there was an agreement to accomplish the unlawful act, with the intent to accomplish the act

108
Q

A woman went to visit her elderly grandfather, who suffered from severe dementia and numerous physical ailments. The grandfather had great difficulty walking, and was largely confined to a wheelchair. The woman had always been scared of her grandfather because he had always had a short temper and a violent streak. At one point during their visit, the grandfather became confused and seemed to think that the woman was a girlfriend from his youth. The grandfather told the woman that he knew she had cheated on him, and that he ought to get the gun out of his dresser and shoot her. The woman quickly ran to the dresser and grabbed the gun. The grandfather threatened that he would take the gun and shoot her, at which point the woman shot her grandfather, killing him instantly. The woman was charged with homicide offenses, and the woman claimed self-defense. The evidence at trial showed that she honestly believed her grandfather would kill her, although this belief was unreasonable given the grandfather’s lack of mobility.

Is the woman likely to be convicted of any homicide offense?

A) The woman may be convicted of second-degree murder.
B) The woman may be convicted of voluntary manslaughter.
C) The woman may be convicted of involuntary manslaughter.
D) The woman is unlikely to be convicted of homicide.

A

B) The woman may be convicted of voluntary manslaughter.

The woman would likely be convicted of voluntary manslaughter under the imperfect self-defense rule.

Imperfect self-defense occurs when the person claiming self-defense unjustifiably kills her attacker, such as when she honestly but unreasonably believes self-defense is required. The rule reduces the charge from murder to voluntary manslaughter.

109
Q

A defendant is on trial for a series of rapes. Based on victim accounts, the rapist would show up at the victims’ homes with a bouquet of yellow roses blocking his face, and then spray the victims in the face with mace to prevent them from seeing his face. All of the victims were attacked after providing their personal information to the same profile on a dating site, a profile later linked to the defendant. After obtaining a valid search warrant for the defendant’s home, the police searched the home but found no incriminating evidence. The police then searched the backyard, which abutted a public road and was not enclosed by a fence. They discovered a large plot of yellow roses along the road approximately 50 yards away from the home. Upon closer inspection, the police discovered that some of the rose bushes were adorned with personal trinkets reported as missing from each of the rapist’s victims.

Can the defendant successfully move to exclude evidence of the trinkets?

A) No, because the incriminating character of the yellow roses was immediately apparent, and the officers were in the back yard for a lawful purpose.
B) No, because there was no reasonable expectation of privacy in the rose garden.
C) Yes, because the rose garden was protected as part of the curtilage of the home.
D) Yes, because the search warrant obtained by the police only encompassed a search of the home.

A

B) No, because there was no reasonable expectation of privacy in the rose garden.

Here, the rose garden was in the backyard along a public road and approximately 50 yards from the home. The backyard was not enclosed by a fence. There is no evidence that the defendant took steps to protect the rose garden adorned with trinkets from observation by the public, so it is not protected as a part of the curtilage of his home.

Note: (A) is incorrect because it states the standard for what may be seized in plain view in private areas, such as a home. Here, the rose garden is in an open field along a public road. Therefore, the incriminating character of the roses is irrelevant.

110
Q

A defendant was charged with perjury by an information filed by a state prosecutor. The offense is punishable by imprisonment of up to three years. The defendant sought to waive his right to a jury trial. However, the prosecution objected to this waiver.

Is the court constitutionally required to permit the defendant to waive his right to a jury trial?

A) No, because perjury is punishable by imprisonment of up to three years.
B) No, because a defendant does not have a constitutional right to a bench trial.
C) Yes, because the Sixth Amendment’s Right to a Jury Trial applies to the states through the Fourteenth Amendment.
D) Yes, because the choice of whether to exercise the right to a jury trial rests solely with the defendant.

A

B) No, because a defendant does not have a constitutional right to a bench trial.

While a defendant may waive the right to a jury trial and opt for a trial by judge, known as a “bench trial,” a defendant does not have a constitutional right to a bench trial.

Note: The Sixth Amendment Right to a Jury Trial does apply to the states through the Fourteenth Amendment, but the right to a jury trial does not include the absolute right to waive a jury trial.

111
Q

On trial for second-degree murder, the defendant pled not guilty, asserting that he acted in self-defense.

The statute in the jurisdiction defines second-degree murder as “the intentional killing of another human being with malice and without provocation.” The jurisdiction also has a statute stating that “all affirmative defenses are to be proved by the defense, and the burden of persuasion shall be by a preponderance of the evidence.”

The judge instructed the jury that the self-defense evidence presented by the defendant should not be considered by the jury for any purpose unless the jury first determined that this evidence satisfied the “preponderance of the evidence” standard. The jury found the defendant guilty of second-degree murder.

The defendant appealed his conviction, contending that the state statute and the jury instructions violated his constitutional rights. How should the appellate court rule on this appeal?

A) Both the statute and jury instructions violate the Due Process Clause of the Fourteenth Amendment.
B) The statute violates the Due Process Clause of the Fourteenth Amendment, but the jury instructions do not.
C) The jury instructions violate the Due Process Clause of the Fourteenth Amendment, but the statute does not.
D) Neither the statute nor the jury instructions violate the Due Process Clause of the Fourteenth Amendment.

A

C) The jury instructions violate the Due Process Clause of the Fourteenth Amendment, but the statute does not.

The jury instructions improperly prevent the jury from considering the defendant’s self-defense evidence, not as an affirmative defense, but as a defense to the elements of the crime that the prosecution must prove.

Note: A state (or the federal government) may place the burden of proving an affirmative defense, such as self-defense, on the defendant without violating the Due Process Clause. The preponderance of the evidence standard for judging whether the defendant has met this burden is also constitutional.

112
Q

A defendant was convicted of aggravated assault. His attorney appealed the conviction, and the first verdict was overturned based on the judge’s finding that after viewing the evidence in the light most favorable to the prosecution, no reasonable fact finder could have voted to convict. The prosecution now seeks to retry the defendant for the same crime. The defendant’s attorney objects to the retrial on double jeopardy grounds.

Should the retrial be allowed?

A) Yes, because a defendant waives any double jeopardy rights by filing an appeal of his conviction.
B) Yes, because the Double Jeopardy Clause only prevents retrial when a defendant has been acquitted or when the prosecution dismisses its case.
C) No, because retrial after reversal of a conviction due to insufficiency of evidence is barred by the Double Jeopardy Clause.
D) No, because retrial after a reversal of a conviction on any grounds is prohibited by the Double Jeopardy Clause.

A

C) No, because retrial after reversal of a conviction due to insufficiency of evidence is barred by the Double Jeopardy Clause.

When a conviction is reversed on these grounds, jeopardy is implicated, and the prosecution is barred from retrying the defendant for the same offense.

Note: Double Jeopardy also applies to retrial of cases overturned due to insufficiency of evidence or if the prosecution asks for a mistrial because of an inability to locate a witness

113
Q

A law makes it a crime to “knowingly sell, distribute, or barter a sexually explicit film featuring actors younger than the age of majority.” The owner of an adult video store sold explicit videos in her store that featured 18-year-old actors, but she took reasonable steps to ensure that no videos featuring younger actors were sold in her store. The video store owner, however, incorrectly believed that the age of majority in the jurisdiction was 18; in fact, the age of majority was 19 years old. The owner was arrested and charged with violating the statute in a jurisdiction that has adopted the Model Penal Code. The prosecution does not contest that her error was made honestly. Should she nonetheless be convicted?

A) Yes, because the owner’s error was a mistake of law, which is not a valid defense.
B) Yes, because the owner knowingly sold the illegal videos.
C) No, because the owner’s error negated the requisite mens rea.
D) No, because the owner’s conscious objective was not to engage in selling the illicit videos.

A

C) No, because the owner’s error negated the requisite mens rea.

If a statute does not state the culpable mind applicable to all material elements of the crime, then the mens rea applicable to one material element is applicable to all material elements unless a contrary purpose plainly appears.

MPC: “knowingly” state of mind is applied to each and every element “sell, distribute, or barter a sexually explicit film” and “featuring actors younger than the age of majority” element.

Here, the store owner subjectively did not know that the videos she was selling featured performers below the age of majority; thus, she cannot be convicted under the statute

Note: an honest mistake of law can be a valid defense when the mistake negates the required intent

114
Q

A man decided to master the art of throwing knives. He practiced for several years, until he had perfected his skills and was able to hit a spot no larger than a dime with confidence. After demonstrating his prowess to a friend, the man convinced the friend to stand against a wall while the man threw knives at her. The man threw three knives extremely close to the friend, but the fourth knife struck the friend, injuring her slightly. Although the friend’s injury was minor, unbeknownst to the man, she had a rare blood disorder that caused her to bleed to death. The crimes below are listed in ascending order of seriousness. What is the most serious common law crime for which the man can be convicted?

Answers:

A) Battery
B) Involuntary manslaughter
C) Voluntary manslaughter
D) Murder

A

D) Murder

The man may be convicted of depraved heart murder: killing that results when the defendant acts with reckless indifference to a known and unjustifiably high risk. For this type of murder, the man need not have had the intent to cause either death or serious bodily injury.

115
Q

The driver of a pickup truck, through no fault of his own, struck a pedestrian who darted into the street in front of the driver’s vehicle. The pedestrian was thrown up into the air and died instantly when her head struck the pavement of the street. The driver, panicked by the event, stepped on the accelerator pedal. The body of the pedestrian became lodged underneath the truck. The driver drove for a city block before abandoning the truck and fleeing on foot. Under state law, fleeing the scene of an accident is a felony.

Can the driver be charged with involuntary manslaughter?

A) No, because the driver acted in the heat of the moment without malice.
B) No, because the pedestrian died when her head struck the pavement.
C) Yes, because the driver was criminally negligent in driving for a city block after the striking the pedestrian.
D) Yes, because the driver was guilty of a felony.

A

B) No, because the pedestrian died when her head struck the pavement.

Causation issue: for a defendant to be guilty of involuntary manslaughter, there must be a causal connection between the unlawful act and the death. Here, although the pedestrian did die, and her death would not have occurred but for the pedestrian having been struck by the pickup truck, the facts provide that the driver was not legally responsible for that collision.

Note: The driver’s conduct in continuing to drive for a city block after striking the pedestrian certainly can be construed as criminal negligence. However, he acted in this manner after the pedestrian had already died through no fault of the driver’s. Consequently, the driver’s criminal negligence was not the cause of the pedestrian’s death, and will not support a charge of involuntary manslaughter.

116
Q

A husband and his friend agreed to kill the husband’s wife and split the insurance proceeds the man received following the wife’s death. The friend helped the husband obtain a handgun to commit the murder, but on the night of the planned murder, the friend decided that she could not go through with it. Afraid that the husband would kill her if she went to the police, she did not contact them. Instead, she called the husband and told him that she could not go through with the crime and urged him not to kill his wife. The husband ignored her and went through with the killing. The husband and friend were subsequently arrested and the friend was charged with conspiracy to commit murder. Should the friend be convicted?

A) No, because she successfully withdrew from the conspiracy.
B) No, because at the time of the murder, the friend lacked the intent for the wife to be killed.
C) Yes, because the friend committed an overt act in furtherance of the conspiracy.
D) Yes, because withdrawal is not a valid defense to conspiracy.

A

D) Yes, because withdrawal is not a valid defense to conspiracy.

Majority: the crime is complete at the moment an overt act in furtherance of the conspiracy has been completed.

CL: At common law, the conspiracy is complete as soon as the parties enter into the agreement.

WITHDRAWAL TO CONSPIRACY NOT A DEFENSE ONCE AN AGREEMENT OR OVERT ACT IS MADE

117
Q

A witness who was not a defendant invoked his Fifth Amendment right to remain silent during a federal criminal trial for insider trading. After being given derivative-use immunity, the witness testified. Several weeks later, the witness was a defendant in a state-law civil fraud proceeding based on his previous testimony in the federal trial. He moved to dismiss the case on the grounds that the previous grant of immunity protected him against a future action against him. Will the defendant’s motion be granted?

A) Yes, because a grant of immunity can be given to a witness who is not a defendant.
B) Yes, because the defendant was given derivative-use immunity.
C) No, because the defendant’s immunity was limited to federal prosecution.
D) No, because the defendant’s immunity does not extend to a subsequent civil trial.

Derivative-use immunity protects a witness from the use of the witness’s own testimony, or any evidence derived from that testimony, against the witness in a subsequent prosecution, but does not protect him from its use in a civil suit.

A

D) No, because the defendant’s immunity does not extend to a subsequent civil trial.

Derivative-use immunity protects a witness from the use of the witness’s own testimony, or any evidence derived from that testimony, against the witness in a subsequent prosecution but does not protect him from its use in a civil suit.

Note: Immunity, whether transactional or derivative-use immunity, does not apply to subsequent civil actions.

118
Q

A 15-year-old male was being tried in state court as an adult for murder. At voir dire, the prosecutor exercised all of his peremptory challenges to exclude persons under the age of 30 from the jury. The defendant’s attorney timely raised the issue as to whether the prosecutor had utilized his peremptory challenges in an unconstitutional manner. In response to questioning by the court, the prosecutor stated that it was his intent to exclude persons who, because of their age, would be sympathetic to the defendant. The judge found that the prosecutor’s reason was genuine and not pretextual.

Should the judge sustain the defendant’s objection?

A) Yes, because the prosecutor’s use of peremptory challenges violates the defendant’s Sixth Amendment right to trial by an impartial jury.
B) Yes, because the defendant was a member of the affected class.
C) No, because the prosecutor is permitted to exercise peremptory challenges for any rational reason.
D) No, because the prosecutor’s use of peremptory challenges does not violate the Equal Protection Clause.

A

D) No, because the prosecutor’s use of peremptory challenges does not violate the Equal Protection Clause.

A prosecutor is free to exercise peremptory challenges in any manner he sees fit unless such exercise violates the Equal Protection Clause. The Equal Protection Clause prevents the use of peremptory challenges for racially or gender motivated reasons.

Note: the prosecution, as well as the defense, is constrained by the Equal Protection Clause in its use of peremptory challenges.

119
Q

A defendant was charged with the sale of narcotics. At his trial, the prosecution planned on calling as witnesses the police officer who investigated the crime, an eyewitness to the crime, a desk officer to testify regarding chain of custody, and a former co-defendant who had reached a plea agreement with the prosecution. The defendant demanded, as a matter of right, that each of these individuals be excluded from the courtroom to prevent them from hearing the testimony of the other witnesses. The prosecution objected to removing any of these individuals from the courtroom. Which of the following individuals should the judge order be removed from the courtroom?

A) The eyewitness only.
B) The eyewitness and the former co-defendant only.
C) The eyewitness, the former co-defendant, and the desk officer, but not the investigating officer.
D) The eyewitness, the former co-defendant, the desk officer, and the investigating officer.

A

C) The eyewitness, the former co-defendant, and the desk officer, but not the investigating officer

At a party’s request, the court must order the exclusion of a witness from the courtroom so that the party cannot hear the testimony of the other witnesses, unless an exception applies. There is an exception for an officer or employee of a party who is not a natural person, and this exception has frequently been applied to the police officer in charge of investigating a criminal case.

120
Q

A man became intoxicated after drinking at a neighborhood bar for several hours. He left the bar and went to a party at a friend’s house, where he struck up a conversation with a woman at the party. After a few minutes, the man grabbed the woman’s arm, pulled her into an empty room, and attempted to have sexual intercourse with her. The woman struggled with the man, and, before intercourse occurred, was able to break free and exit the room. The man was arrested and charged with attempted rape. At trial, the man testified that at the time of the incident he believed that the woman had consented to sexual intercourse with him. If the jury believes the man’s testimony, should he be convicted?

A) No, because the man’s intoxication prevented him from understanding the wrongfulness of his act.
B) No, because the man believed that the woman had consented to intercourse.
C) Yes, because the man’s intoxication was voluntary.
D) Yes, because rape is a crime of malice and intoxication is not a defense to malice crimes.

A

B) No, because the man believed that the woman had consented to intercourse.

Rape = general intent crime

Attempted rape = specific intent crime (Inchoate)

Intoxication can negate mens rea for a specific intent crime

121
Q

A woman met a man at a party at the home of a third person. The woman noticed that the man was wearing an expensive gold watch. As the party was winding down and the woman and man were alone, the woman slipped a sedative into the man’s drink. Waiting until the man passed out, the woman then removed the watch from the man’s wrist and left the party. Later, the party’s host discovered the man asleep, and revived him. When the man discovered that his watch was missing, the man called the police. The man, who lived at home with his parents, had taken the watch from his father’s dresser for the evening, without his father’s permission. The woman was arrested and charged with robbery. Can she be convicted of the crime?

A) Yes, because the woman used force to permanently deprive the man of the watch he was wearing.
B) Yes, because the taking took place at a dwelling.
C) No, because the watch belonged to the man’s father and the man did not have permission to use it.
D) No, because the man was unconscious when his watch was taken.

A

A) Yes, because the woman used force to permanently deprive the man of the watch he was wearing.

Robbery:
(i) larceny
(ii) from the person or presence of the victim
(iii) by force or intimidation.

The force used by a defendant must be more than the amount necessary to effectuate taking and carrying away the property. Force can include giving a victim drugs in order to induce unconsciousness and thereby permit the larceny to occur.

122
Q

A woman took her car to an unscrupulous auto mechanic’s garage for a tune-up. The woman’s car had a new and expensive set of tires that the mechanic coveted. The woman left her car at the garage overnight. Later that night, after the woman had left the premises, the mechanic took the tires off the woman’s car, put them into a back room of his garage, and replaced the tires with a cheap, old set. That same evening, the woman’s friend told her about the mechanic’s unscrupulous nature, and that he had a habit of stealing tires. The woman went back to the garage the next morning. Noticing that the tires on her vehicle were different, she demanded that the new, expensive tires be put back on the vehicle. The mechanic complied, and the woman left the premises. The woman reported the mechanic to the police, and the mechanic is charged with larceny.

Based on the foregoing facts, should he be convicted of the crime?

A) Yes, because the mechanic moved the tires from the car to the back room.
B) Yes, because the mechanic had a present intent to permanently deprive the woman of the tires.
C) No, because the car was left with the mechanic by consent.
D) No, because the tires were returned to the woman before she was permanently deprived of them.

A

C) No, because the car was left with the mechanic by consent.

Larcency: the initial taking and asportation of another’s property must be trespassory; that is, the defendant must not be legally entrusted with the property.

Here, the woman entrusted the mechanic with her vehicle (and the tires on the vehicle). Thus, the initial taking of the tires was not trespassory, and the mechanic’s crime was embezzlement, not larceny

123
Q

A man asked a friend to burn down the man’s residence so the man could collect the fire insurance proceeds. The friend stated that she would be willing to set fire to the residence for $20,000. The man offered $10,000, but the friend refused. Later, the man set fire to an office building that he owned in order to collect the fire insurance proceeds. The man honestly, but unreasonably and incorrectly, believed that there was no one in the building when he set the fire. There was a person in the office building at the time of the fire who escaped unharmed. The man is charged with solicitation and arson. The relevant statute defines arson as “the malicious burning of any dwelling or occupied structure.” Can the man be convicted of these crimes?

A) No, as to both solicitation and arson.
B) Yes, as to both solicitation and arson.
C) Yes, as to arson only.
D) Yes, as to solicitation only.

A

B) Yes, as to both solicitation and arson.

Solicitation = merely asking/inviting is the crime itself
If the other person agrees = conspiracy
If the other person commits the offense = completed offense
If the person does not agree = solicitation

The man can be convicted of arson because he burned down his office building at a time when a person was inside. The fact that the person inside was not harmed is irrelevant.

Note: The man’s unreasonable mistake as to the person’s presence in the building is not a defense to a malice crime.

124
Q

A defendant is on trial for cocaine possession. The cocaine was found during a warrantless search of the defendant’s car by a police officer. The search occurred immediately after the defendant was arrested for driving a car with an inoperative taillight, a misdemeanor punishable only by a fine. The defendant had been placed in a police car prior to the search. The cocaine was found inside a closed bag on the back seat of the passenger compartment of the defendant’s car. The defendant now moves to suppress the cocaine. Will the defendant’s motion be granted?

A) Yes, because the defendant was in the police car at the time of the search.
B) Yes, because the arrest was unreasonable and the cocaine seized was a fruit of the poisonous tree.
C) No, because the police may search a car without a warrant under the automobile exception.
D) No, because the search was a lawful search incident to arrest.

A

A) Yes, because the defendant was in the police car at the time of the search.

Lawful search incident to arrest = a search of a car in which the defendant was an occupant must be made at the time that the defendant has access to the car or to uncover evidence of the crime for which the defendant was arrested.

Note: a police officer’s arrest of an individual for a misdemeanor that is punishable only by a fine is not unreasonable under the Fourth Amendment.

125
Q

At about 5:00 a.m., firefighters responded to a house fire. The fire was brought under control in about an hour and completely extinguished approximately two hours later. In the early afternoon, a city fire inspector came to the house with an administrative warrant. The warrant authorized the inspector to search for the cause of the fire and seize items related to it. The inspector first searched the basement of the house, finding a barely recognizable electric curling iron plugged into a partially melted timer, which was set to turn on at about 4:00 a.m. In addition, the inspector found the iron in a large soot blackened tub. Nearby were several empty gallon containers labeled “turpentine.” The inspector seized the iron, timer, tub and empty turpentine containers, believing that they constituted evidence necessary to establish the cause of the fire. While the ground level of the house was almost completely destroyed by the fire, the second level was in much better condition. On the second floor, the inspector noticed that there were several empty frames, their pictures apparently having been removed, hanging on the walls. Also the inspector noted that none of the three upstairs bedrooms contained electronic equipment, such as televisions or computers, despite the presence of empty power strips plugged into outlets in these rooms. The inspector seized the empty frames and power strips, believing that they constituted evidence the homeowner had deliberately set the fire. The family who lived in the house was away on vacation at the time of the fire and did not return until the following day. The homeowner was charged with arson. Can the homeowner successfully object to the introduction into evidence of the items seized by the inspector from the second floor of the house?

A) No, because the items were seized by a fire inspector rather than a police officer.
B) No, because the search was conducted under exigent circumstances.
C) No, because the search was conducted pursuant to an administrative warrant.
D) Yes, because the seizure of these items exceed the scope of the warrant.

A

D) Yes, because the seizure of these items exceed the scope of the warrant.

Although the warrant authorized the fire inspector to search for the cause of the fire, the fire inspector determined the cause of the fire in his search of the basement.

As a result, his search of the second floor of the house exceeded the scope of the administrative warrant. On the second floor, he was searching for and found evidence related to whether the homeowner had deliberately set the fire =** whether the homeowner was guilty of arson, rather than the cause of the fire*

126
Q

A man was arrested and charged with stealing $5 million in an armed bank robbery. The police gave the man his Miranda warnings, and the man indicated that he refused to speak to the police without his lawyer present. The police then placed the man in a holding cell in the police station with an undercover police officer who was posing as a man arrested for drunken driving. The undercover officer asked the man several questions in an attempt to elicit information about the crime and discover where the man’s accomplices might be hiding. Believing that he was talking to another prisoner, the man made several self-incriminating statements about the bank robbery, which were secretly taped by a “bug” planted in the holding cell by the police. Assume that at a subsequent trial for the bank robbery, the defense makes a motion to suppress all evidence regarding the statements the man made in the holding cell to the undercover officer. How should the court rule?

A) Deny the motion because the man had no expectation of privacy in a police holding cell.
B) Deny the motion because the man waived any right to counsel by voluntarily speaking to the undercover officer.
C) Grant the motion because the police violated the man’s Sixth Amendment right to counsel.
D) Grant the motion because the evidence constitutes the fruit of an illegal search violating the man’s Fourth Amendment rights.

A

C) Grant the motion because the police violated the man’s Sixth Amendment right to counsel.

The man’s Sixth Amendment right to counsel was violated by the police action. Under the Sixth Amendment, interrogation constitutes a “critical stage” of prosecution, such that a post-charge interrogation in the absence of counsel violates a defendant’s Sixth Amendment right to counsel unless the defendant has waived such right.

Interrogation= formal questioning by the police AND any conduct of the police that is intended to elicit a response from a defendant.

Note: expectation of privacy in his cell is a 4th Amendment issue, relating to search and seizure